Conflict of Laws

36
Gibbs vs. Gov’t. of PI (1933) FACTS: Spouses Allison & Eva were residents & citizens of California owning parcels of land in Manila. Eva died. Allison, as administrator files petition to declare lands in his favor pursuant to California law. California Civil Code provides that upon wife’s death previous to husband, community property belongs absolutely to husband. HELD: California Civil Code will not apply. The law of the place where land is situated governs its descent, alienation &transfer & for the effect & construction of wills & other conveyances. As mandated by Philippine law, the lands were acquired as community property in the conjugal partnership. The wife was vested with a title equal to that of her husband. Upon her death, if there are no obligations of the decedent, her share in the conjugal property is transmitted to the heirs by succession. [G.R. No. 75242. September 2, 1992.] MANILA RESOURCE DEVELOPMENT CORPORATION, Petitioner, v. THE NATIONAL LABOR RELATIONS COMMISSION and RUBEN MANAHAN, Respondents. Ceferino Padua Law Office for Petitioner. Rafael Y. Viola for Respondents. SYLLABUS 1. LABOR AND SOCIAL LEGISLATIONS; ACT NO. 3110; NOT APPLICABLE TO RECORDS DESTROYED BY FIRE IN CASES PENDING BEFORE THE NATIONAL LABOR RELATIONS COMMISSION; TECHNICAL RULES NOT BINDING IN LABOR CASES. — Act No. 3110 which took effect on March 19, 1923, and which deals with the procedure of reconstituting records of pending judicial proceedings destroyed by fire, strictly speaking, has no application to records destroyed by fire in cases pending with the NLRC, a quasi-judicial body. Act No. 3110 as its Title indicates refers to "the reconstruction of the records of pending judicial proceedings. . . ." Moreover, Article 221 of the Labor Code, as amended, provides that findings of facts in labor cases should be speedily ascertained without regard to technicalities of law or procedure all in the interest of due process. 2. ID.; EMPLOYMENT AGENCY; SOLIDARILY LIABLE WITH THE FOREIGN BASED EMPLOYER FOR ANY VIOLATIONS OF THE EMPLOYMENT CONTRACT AND RECRUITMENT AGREEMENT. — We find no need to rule on the constitutionality of Section 10(a)(2), Rule V, Book I of the Omnibus Rules to Implement the Labor Code. A similar question — though not under the guise of the unconstitutionality of said rule — cropped up in Royal Crown International v. NLRC, (178 SCRA 569) There, We ruled that:" [P]etitioner conveniently overlooks the fact that it had voluntarily assumed solidary liability under the various contractual undertakings it submitted to the Bureau of Employment Service . . . It was also required to file with the Bureau a formal appointment of agency contract executed by the foreign based employer in its favor to recruit and hire personnel for the former, which contained a provision empowering it to sue and be sued jointly and solidarily with the foreign principal for any of the violations of the recruitment agreement and the contracts of employment [Section 10(a) (2), Rule V, Book I of the Rules to Implement the Labor Code (1976)]. Petitioner was required as well to post such cash and surety bonds as determined by the Secretary of Labor to guarantee compliance with prescribed recruitment procedures, rules and regulations, and terms and conditions of employment as appropriate [Section 1 of Pres. Dec. 1412 (1978) amending Article 31 of the Labor Code]. These contractual undertakings constitute the legal basis for holding petitioner, and other private employment or recruitment agencies, liable jointly and severally with its principal, the foreign based employer, for all claims filed by recruited workers which may arise in connection with the implementation of the service agreements or employment contracts . . . 3. ID.; ID.; FINDINGS OF FACT BY THE NLRC ARE GENERALLY BINDING ON APPEAL. — Regarding petitioner’s other minor arguments, the first of which is whether or not the foreign employer, OBALCO, could dismiss private respondent for misrepresenting his qualifications, plus the fact that such dismissal was allegedly in line with the three-month probationary period embodied in the supplemental agreement private respondent signed with his foreign employer upon arrival in Saudi Arabia, this matter has already been disposed of in the WAAO, POEA decision and affirmed by the NLRC, whose findings of fact are binding on Us, absent the exceptions which do not obtain in this case. 4. CIVIL LAW; PARTY AUTONOMY IN CONTRACTS, NOT AN ABSOLUTE PRINCIPLE; PARTIES NOT AT LIBERTY TO CONTRACT AWAY PROVISIONS OF LAW HEAVILY IMPRESSED WITH PUBLIC INTEREST. — As to petitioner’s argument over the primacy of the labor laws of Saudi Arabia over Philippine labor laws, suffice it to say that in the Pakistan International Airlines (Pakistan International Airlines Corporation v. Ople, 190 SCRA 99, 103) case We had already ruled that Philippine laws and regulations cannot be rendered illusory by the parties agreeing on some other laws to govern their relationship. "As We stated in said case: A contract freely entered into should, of course, be respected, as PIA argues, since a contract is the law between the parties. [Henson v. Intermediate Appellate Court, 148 SCRA 11 (1987)] The principle of party autonomy in contracts is not, however, an absolute principle. The rule in Article 1306, of our Civil Code is that the contracting parties may establish such stipulations as they may deem convenient, ‘provided they are not contrary to law, morals, good customs, public order or public policy.’ Thus, counter-balancing the principle of autonomy of contracting parties is the equally general rule that provisions of

description

compilation of internet digests

Transcript of Conflict of Laws

Gibbs vs. Gov’t. of PI

(1933)

FACTS: Spouses Allison & Eva were residents & citizens of California owning parcels of land in Manila. Eva died. Allison, as administrator files petition to declare lands in his favor pursuant to California law. California Civil Code provides that upon wife’s death previous to husband, community property belongs absolutely to husband.

HELD: California Civil Code will not apply. The law of the place where land is situated governs its descent, alienation &transfer & for the effect & construc-tion of wills & other conveyances.

As mandated by Philippine law, the lands were acquired as community property in the conjugal partnership. The wife was vested with a title equal to that of her husband. Upon her death, if there are no obligations of the decedent, her share in the conjugal property is transmitted to the heirs by succession.

[G.R. No. 75242. September 2, 1992.]

MANILA RESOURCE DEVELOPMENT CORPORATION, Petitioner, v. THE NATIONAL LABOR RELATIONS COMMISSION and RUBEN MANAHAN, Respondents.

Ceferino Padua Law Office for Petitioner.

Rafael Y. Viola for Respondents.

SYLLABUS

1. LABOR AND SOCIAL LEGISLATIONS; ACT NO. 3110; NOT APPLICABLE TO RECORDS DESTROYED BY FIRE IN CASES PENDING BEFORE THE NATIONAL LABOR RELATIONS COMMISSION; TECHNICAL RULES NOT BINDING IN LABOR CASES. — Act No. 3110 which took effect on March 19, 1923, and which deals with the procedure of reconstituting records of pending judicial proceedings destroyed by fire, strictly speaking, has no application to records destroyed by fire in cases pending with the NLRC, a quasi-judicial body. Act No. 3110 as its Title indicates refers to "the reconstruction of the records of pending judicial proceedings. . . ." Moreover, Article 221 of the Labor Code, as amended, provides that findings of facts in labor cases should be speedily ascertained without regard to technicalities of law or procedure all in the interest of due process.

2. ID.; EMPLOYMENT AGENCY; SOLIDARILY LIABLE WITH THE FOREIGN BASED EMPLOYER FOR ANY VIOLATIONS OF THE EMPLOYMENT CONTRACT AND RECRUITMENT AGREEMENT. — We find no need to rule on the constitutionality of Section 10(a)(2), Rule V, Book I of the Omnibus Rules to Implement the Labor Code. A similar question — though not under the guise of the unconstitutionality of said rule — cropped up in Royal Crown International v. NLRC, (178 SCRA 569) There, We ruled that:" [P]etitioner conveniently overlooks the fact that it had voluntarily assumed solidary liability under the various contractual undertakings it submitted to the Bureau of Employment Service . . . It was also required to file with the Bureau a formal ap -pointment of agency contract executed by the foreign based employer in its favor to recruit and hire personnel for the former, which contained a provision empowering it to sue and be sued jointly and solidarily with the foreign principal for any of the violations of the recruitment agreement and the contracts of employment [Section 10(a) (2), Rule V, Book I of the Rules to Implement the Labor Code (1976)]. Petitioner was required as well to post such cash and surety bonds as determined by the Secretary of Labor to guarantee compliance with prescribed recruitment procedures, rules and regulations, and terms and conditions of employment as appropriate [Section 1 of Pres. Dec. 1412 (1978) amending Article 31 of the Labor Code]. These contractual undertakings constitute the legal basis for holding petitioner, and other private employment or recruitment agencies, liable jointly and severally with its principal, the for-eign based employer, for all claims filed by recruited workers which may arise in connection with the implementation of the service agreements or employ-ment contracts . . .

3. ID.; ID.; FINDINGS OF FACT BY THE NLRC ARE GENERALLY BINDING ON APPEAL. — Regarding petitioner’s other minor arguments, the first of which is whether or not the foreign employer, OBALCO, could dismiss private respondent for misrepresenting his qualifications, plus the fact that such dis -missal was allegedly in line with the three-month probationary period embodied in the supplemental agreement private respondent signed with his foreign employer upon arrival in Saudi Arabia, this matter has already been disposed of in the WAAO, POEA decision and affirmed by the NLRC, whose findings of fact are binding on Us, absent the exceptions which do not obtain in this case.

4. CIVIL LAW; PARTY AUTONOMY IN CONTRACTS, NOT AN ABSOLUTE PRINCIPLE; PARTIES NOT AT LIBERTY TO CONTRACT AWAY PROVI -SIONS OF LAW HEAVILY IMPRESSED WITH PUBLIC INTEREST. — As to petitioner’s argument over the primacy of the labor laws of Saudi Arabia over Philippine labor laws, suffice it to say that in the Pakistan International Airlines (Pakistan International Airlines Corporation v. Ople, 190 SCRA 99, 103) case We had already ruled that Philippine laws and regulations cannot be rendered illusory by the parties agreeing on some other laws to govern their relation -ship. "As We stated in said case: A contract freely entered into should, of course, be respected, as PIA argues, since a contract is the law between the par-ties. [Henson v. Intermediate Appellate Court, 148 SCRA 11 (1987)] The principle of party autonomy in contracts is not, however, an absolute principle. The rule in Article 1306, of our Civil Code is that the contracting parties may establish such stipulations as they may deem convenient, ‘provided they are not contrary to law, morals, good customs, public order or public policy.’ Thus, counter-balancing the principle of autonomy of contracting parties is the equally general rule that provisions of applicable law, especially provisions relating to matters affected with public policy are deemed written into the contract. [Com-missioner of Internal Revenue v. United Lines Co., 5 SCRA 175 (1962)]. Put a little differently, the governing principle is that parties may not contract away applicable provisions of law especially peremptory provisions dealing with matters heavily impressed with public interest. The law relating to labor and em-ployment is clearly such an area and parties are not at liberty to insulate themselves and their relationships from the impact of labor laws and regulations by simply contracting with each other. It is thus necessary to appraise the contractual provisions invoked by petitioner PIA in terms of their consistency with ap-plicable Philippine law and regulations."cralaw virtua1aw library

5. LABOR AND SOCIAL LEGISLATIONS; REQUIREMENTS OF DUE PROCESS; SUBSTANTIALLY COMPLIED WITH IN CASE AT BAR. — As to peti-tioner’s complaint of denial of due process, settled is the rule that submission of position papers and memoranda in labor cases fulfills the requirements of due process. In the case at bar, aside from filing a position paper with the WAAO, POEA, petitioner filed a Memorandum of Appeal in the NLRC, aside from numerous other pleadings. Since the right to be heard was accorded petitioner, it cannot now claim that it was denied due process in the resolution of its case.

D E C I S I O N

NOCON, J.:

Petitioner Manila Resource Development Corporation questions the Jurisdiction of public respondent National Labor Relations Commission in issuing its Decision 1 promulgated on July 3, 1986 affirming with modification the Decision 2 promulgated on April 22, 1983 by public respondent Workers Assistance and Adjudication Office, Philippine Overseas Employment Administration.

As found by the public respondent NLRC, the antecedents or this case are as follows:cralawnad

"It appears that sometime in June 1981, the complainant applied for overseas employment with respondent Manila Resource Development Corporation (MANRED) and after he was interviewed, he was directed to submit his bio-data, which he did. Upon receipt by MANRED of an advice from its principal, Obaid & Al Mulla Construction Company (OBALCO) to the effect that the latter needed, among others, a Mechanical Engineer with at least ten year ’s expe-rience in sheeting ventilation, heavy duty air-conditioning and piping and duct works installation, the complainant was summoned for another interview at which time the telex message from OBALCO was allegedly shown to him and he allegedly gave full assurance that he was qualified for the position. He was allegedly required again to submit a bio-data stating therein his qualifications and experience that would meet the skill and experience required by the job order of OBALCO. He also allegedly submitted another bio-data wherein he unqualifiedly declared that he had all the qualifications and experience called for but, according to the respondents, his record folder was misplaced and could not be found. On the basis of his representation concerning his work experience and qualifications, an employment contract dated 5 August 1981 was signed by him and Wilfredo V. Fernandez, General Manager of MANRED. The contract provided that the complainant was hired as a Mechanical Engineer for a basic monthly pay of US$1,000.00 with a duration of two years from the date of his departure for Saudi Arabia.

"On 27 January 1982, the complainant left Manila and arrived in Riyadh, Saudi Arabia on 28 January 1982. He reported for duty at OBALCO on 30 January 1982 and was given a final examination concerning his qualifications and experience before starting to work. The respondents claim that by his own verbal and written admission, he failed to meet the qualifications and experiences needed for the work. In a memorandum dated 30 January 1982, it was stated that the summary of work experience indicated in his hand written bio-data were not useful or suitable for the mechanical engineering work at the Royal Terminal. On the same day, he was advised that OBALCO was not willing to employ him for US$1,000.00 per month but he would be retained if he was willing to accept US$360.00 per month as basic pay. He pleaded that his salary be as provided in his employment contract or reduced by 20% only but this was turned down. Consequently, he was repatriated to Manila on 14 February 1982." 3

Consequently, private respondent filed a complaint with the Bureau of Employment Services against petitioner and its General Manager, Wilfredo Fernan -dez, Sr., and consequently amended the same to include OBAID AL MULLA CONSTRUCTION COMPANY. 4

The Bureau of Employment Services, now renamed Workers’ Assistance and Adjudication Office and placed under the Philippine Overseas Employment Administration, rendered a Decision on April 22, 1983 which in its dispositive portion reads as follows:jgc:chanrobles.com.ph

"IN VIEW OF THE FOREGOING, decision is hereby made and entered in favor of complainant ordering respondents Manila Resource Development Corpo-ration and/or Wilfredo Fernandez and its foreign principal, Obaid and Al Mulla Construction Company Riyadh, Saudi Arabia, jointly and solidarily liable to pay complainant, thru this Office within ten (10) days from receipt hereof. the sum of TWENTY FOUR THOUSAND US DOLLARS (US$24,000.00) or its equivalent in Philippine Currency at the current, exchange rate, representing complainant’s unexpired portion of his contract for twenty four (24) months at US$1,000.00 per month.chanrobles law library

"Respondents are also hereby ordered to pay complainant the sum of FOUR THOUSAND SIX HUNDRED TWENTY TWO (P4,622.00) pesos, representing the amount demanded from and paid by complainant to the Respondent." 5

Petitioner filed its Memorandum of Appeal from said WAAO decision on June 14, 1983, which appeal was docketed as NLRC Case No. 82-108.

Meanwhile, a fire occurred on or about November, 1983 which burned the building housing the NLRC office where the records were kept. 6 In an attempt to reconstitute the records of the instant case, private respondent filed on March 12, 1984 an untitled pleading dated March 11, 1984 furnishing the Commis -sion with a xerox copy of the complete file of the case in his possession in compliance with an Order dated March 7, 1984 of the NLRC Hearing Officer. He alleged that petitioner was also sent copies of the same. 7

Petitioner opposed the untitled pleading of March 11, 1984 claiming it was in effect a petition for reconstitution which did not follow the requirements of Act No. 3110. 8

Nevertheless proceedings continued in the NLRC. On July 3, 1986, public respondent NLRC promulgated the assailed decision, which in its dispositive por -tion states as follows:jgc:chanrobles.com.ph

"WHEREFORE, except for the modification excluding respondent Wilfredo Fernandez from the respondents who are ,jointly and solidarily liable to pay the judgment sum, and the reduction of the award in favor of the complainant to his salaries for twelve (12) months at US$1,000.00 per month, aside from the refund of the P4,622.00 paid by him and the 10% attorney’s fees, the Decision appealed from is hereby affirmed in all other respects." 9

Hence, this petition wherein petitioner claims (1) that the NLRC could not validly decide the instant case as all the original records were burned by fire; (2) that petitioner was denied due process at the WAAO, POEA, in view of the summary nature of the proceeding therein; (3) that there is no clear evidence showing that the petitioner is solidarily liable with the foreign employer; and (4) that the foreign employer could dismiss private respondent as he misrepre-sented his qualifications for the job he was hired to do and he had signed a supplemental employment contract with the foreign employer wherein he agreed to a three-month probationary period at the start of his employment.

On October 6, 1986, this Court issued a Temporary Restraining Order "enjoining the respondents from enforcing the writ of execution or any alias writ of ex-ecution issued in NLRC Case No. 82-108, 10 upon motion by petitioner."cralaw virtua1aw library

On November 5, 1986, petitioner filed a supplemental petition claiming that the supplemental employment contract which private respondent signed in Saudi Arabia upon his arrival specifically stipulates that only Saudi Arabian laws can be invoked by either party in any dispute regarding the same, and that the supplemental employment contract prevails over the contract private respondent signed with petitioner in the Philippines and under the generally ac-cepted principles of international law which form part of the law of the land 11 Saudi Arabian Laws govern.

Private respondent, in his Comment to the Petition and Supplemental Petition, countered that:chanroblesvirtualawlibrary

1. Private respondent certified under oath to the authenticity of the records he submitted to public respondent NLRC pursuant to the agreement of January 11, 1985. 12 While petitioner also agreed to submit its own copies of the case, it never did so. There was, therefore, substantial compliance with the re -quirements of Act No. 3110;

2. There was no denial of due process in the manner the WAAO Director decided the case because he decided the same on April 22, 1983 or 10 months after petitioner submitted its Position Paper on June 28, 1982. 13 The delay, if any, was due to the fact that the WAAO waited for petitioner to submit its al -leged evidence coming from Saudi Arabia, namely: (a) The "undertaking" that private respondent allegedly made in Saudi Arabia before his employer, OBALCO, that he does not know anything about air-conditioning, 14 (b) The "missing" bio-data submitted in 1981 by private respondent and (c) The "Sup-plemental Agreement" allegedly signed by private respondent on January 28, 1982, 15 all of which petitioner never submitted;

3. Petitioner should have made private respondent undergo a trade test before sending him to Saudi Arabia to determine his qualification and competency for the position applied for. And the Saudi employer should have tested him through actual work performance instead of rejecting him alone on the basis of private respondent’s handwritten bio-data;

4. Petitioner never submitted to the WAAO Hearing Officer, nor to the NLRC, the copy of the Supplemental Agreement private respondent was supposed to have signed on January 28, 1982, one day after his arrival at Saudi Arabia, which Supplemental Agreement was supposed to have included a provision that private respondent was only a probationary employee for the first three (3) months of his two (2) year contract with OBALCO. The fact that private respon-dent’s Contract with petitioner was for a two-year period rules out any notion that he was supposedly a probationary employee for three months;

5. Section 10, par (a)(2), Rule V, Book I of the Omnibus Rules implementing the Labor Code is the basis for holding petitioner jointly and severally liable with the foreign employer for violations of the recruitment agreement and the contract of employment.

In his Memorandum, petitioner adds another argument to bolster its case, which is that Section 10, Rule V of the Omnibus Rules is unconstitutional be -cause the said rule is without any statutory basis.

The petition is without merit.

Petitioner’s two main arguments are that (1) public respondent NLRC could not validly promulgate a Decision because the records of the case which were destroyed by fire sometime in November, 1983, were not validly reconstituted in accordance with Act No. 3110 and that (2) Section 10, par. (a)(2), Rule V, Book I of the Omnibus Rules implementing the Labor Code, is unconstitutional for want of proper legislative standards, and, therefore, petitioner is not soli -darily liable with OBALCO for the latter’s violations, if any, of private respondent’s service agreement.

Regarding petitioner’s first main argument, it appears from the Original Record that in a conference held on February 27, 1984, both petitioner and private respondent were advised of the total loss of the record of the case as a result of the fire that razed the third floor of the Phoenix Building where the records were kept. 16

When the parties met again on March 7, 1984, private respondent agreed to submit copies of all the documents in his possession to the hearing officer, copy furnished the petitioner who would comment on said reproduced copies and submit additional documents in its possession within ten (10) working days. The case would then be deemed submitted for decision. 17 Both petitioner and private respondent signed their conformity to said agreement on re -constitution of records.chanrobles.com : virtual law library

Petitioner filed a Manifestation opposing the, apparent reconstitution by private respondent of the case without complying with Act No. 3110. 18 Also, when Private respondent filed on January 14, 1985 a "verified Motion to Admit Reconstructed Records Previously Filed," 19 petitioner opposed the same but still did NOT submit to the NLRC Hearing Officer copies of the documents in its own possession.

At any rate, Act No. 3110 which took effect on March 19, 1923, and which deals with the procedure of reconstituting records of pending judicial proceedings destroyed by fire, strictly speaking, has no application to records destroyed by fire in cases pending with the NLRC, a quasi-judicial body. Act No. 3110 as its Title indicates refers to "the reconstruction of the records of pending judicial proceedings . . ." Moreover, Article 221 of the Labor Code, as amended, pro-vides that findings of facts in labor cases should be speedily ascertained without regard to technicalities of law or procedure all in the interest of due process. 20

The plain fact is that petitioner had all the time from March 7, 1984 to July 3, 1986, the date of promulgation of the assailed decision, within which to submit copies of the documents in its possession, but for reasons, known only to itself, it did not do so. As the Solicitor General states, "It is too late now, after los -ing the case, for petitioner to complain about the record." 21

As to petitioner’s second main argument, We find no need to rule on the constitutionality of Section 10(a)(2),, Rule V, Book I of the Omnibus Rules to Imple -ment the Labor Code. A similar question — though not under the guise of the unconstitutionality of said rule — cropped up in Royal Crown International v. NLRC, 22 There, We ruled that: 23

" [P]etitioner conveniently overlooks the fact that it had voluntarily assumed solidary liability under the various contractual undertakings it submitted to the Bureau of Employment Services. In applying for its license to operate a private employment agency for overseas recruitment and placement, petitioner was required to submit, among others, a document or verified undertaking whereby it assumed all responsibilities for the proper use of its license and the imple-mentation of the contracts of employment with the workers it recruited and deployed for overseas employment [Section 2(e), Rule V, Book I, Rules to Imple -ment the Labor Code (1976)]. It was also required to file with the Bureau a formal appointment of agency contract executed by the foreign based employer in its favor to recruit and hire personnel for the former, which contained a provision empowering it to sue and be sued jointly and solidarily with the foreign principal for any of the violations of the recruitment agreement and the contracts of employment Section 10(a) (2), Rule V, Book I of the Rules to Implement the Labor Code (1976)]. Petitioner was required as well to post such cash and surety bonds as determined by the Secretary of Labor to guarantee compli -ance with prescribed recruitment procedures, rules and regulations, and terms and conditions of employment as appropriate [Section 1 of Pres. Dec. 1412 (1978) amending Article 31 of the Labor Code].

"These contractual undertakings constitute the legal basis for holding petitioner, and other private employment or recruitment agencies, liable jointly and severally with its principal, the foreign based employer, for all claims flied by recruited workers which may arise in connection with the implementation of the service agreements or employment contracts [See Ambraque International Placement and Services v. NLRC, G.R. No. 77970, January 28, 1988, 157 SCRA 431; and other cases cited herein.]

And, as in the above-cited case, petitioner questions the effectivity of said rule when the omnibus rules were allegedly not published in the Official Gazette citing Tañada v. Tuvera. 24

This issue was already resolved by Us in the Royal Crown International case 25 when We held:chanrobles lawlibrary : rednad

" [I]t must be emphasized again that petitioner assumed the obligations and liabilities of a private employment agency by contract. Thus, whether or not this omnibus rules are effective in accordance with Tañada v. Tuvera is an issue, the resolution of which does not render at all nugatory the binding effect upon petitioner of its own contractual undertaking.

The Court, consequently, finds it unnecessary to pass upon . . . the implications of Tañada v. Tuvera on the omnibus rules implementing the Labor Code. . . ."cralaw virtua1aw library

Regarding petitioner’s other minor arguments, the first of which is whether or not the foreign employer, OBALCO, could dismiss private respondent for mis-representing his qualifications, plus the fact that such dismissal was allegedly in line with the three month probationary period embodied in the supplemen -tal agreement private respondent signed with his foreign employer upon arrival in Saudi Arabia, this matter has already been disposed of in the WAAO, POEA decision and affirmed by the NLRC, whose findings of fact are binding on Us, absent the exceptions which do not obtain in this case. 26

As to petitioner’s argument over the primacy of the labor laws of Saudi Arabia over Philippine labor laws, suffice it to say that in the Pakistan International Airlines 27 case We had already ruled that Philippine laws and regulations cannot be rendered illusory by the parties agreeing on some other laws to gov -ern their relationship. As We stated in said case:jgc:chanrobles.com.ph

"A contract freely entered into should, of course, be respected, as PIA argues, since a contract is the law between the parties. [Henson v. Intermediate Ap -pellate Court, 148 SCRA 11 (1987)] The principle of party autonomy in contracts is not, however, an absolute principle. The rule in Article 1306, of our Civil Code is that the contracting parties may establish such stipulations as they may deem convenient, provided they are not contrary to law, morals, good cus-toms, public order or public policy. Thus, counter-balancing the principle of autonomy of contracting parties is the equally general rule that provisions of ap-plicable law, especially provisions relating to matters affected with public policy are deemed written into the contract. [Commissioner of Internal Revenue v. United Lines Co. 5 SCRA 175 (1962)]. Put a little differently, the governing principle is that parties may not contract away applicable provisions of law espe -cially peremptory provisions dealing with matters heavily impressed with public interest. The law relating to labor and employment is clearly such an area and parties are not at liberty to insulate themselves and their relationships from the impact of labor laws and regulations by simply contracting with each other. It is thus necessary to appraise the contractual provisions invoked by petitioner PIA in terms of their consistency with applicable Philippine law and regulations."cralaw virtua1aw library

"x       x       x

"Petitioner PIA cannot take refuge in paragraph 10 of its employment agreement which specifies, firstly, the law of Pakistan as the applicable law of the agreement and, secondly, lays the venue for settlement of any dispute arising out of or in connection with the agreement ‘only [in] courts of Karachi, Pak-

istan.’ The first clause of paragraph 10 cannot be invoked to prevent the application of Philippine labor laws and regulations to the subject matter of this case, i.e., the employer-employee relationship between petitioner PIA and private respondents. We have already pointed out that relationship is much af-fected with public interest and that the otherwise applicable Philippine laws and regulations cannot be rendered illusory by the parties agreeing upon some other law to govern their relationship . . . Finally, and in any event, the petitioner PIA did not undertake to plead and prove the contents of Pakistan law on the matter; it must therefore be presumed that the applicable provisions of the law of Pakistan are the same as the applicable provisions of Philippine law. [Miciano v. Primo, 50 Phil. 867 (1924); Collector of Internal Revenue v. Fisher, 110 Phil. 686 [1961]. 28

Lastly, as to petitioner’s complaint of denial of due process, settled is the rule that submission of position papers and memoranda in labor cases fulfills the requirements of due process. 29 In the case at bar, aside from filing a position paper with the WAAO. POEA, petitioner filed a Memorandum of Appeal in the NLRC, aside from numerous other pleadings. Since the right to be heard was accorded petitioner, it cannot now claim that it was denied due process in the resolution of its case. 30

WHEREFORE, finding no merit in the petition, the same is hereby DISMISSED. The assailed decision of public respondent National Labor Relations Com -mission is AFFIRMED en toto. The Temporary Restraining Order issued last October 6, 1986 is hereby lifted. Costs against petitioner.chanrobles virtual lawlibrary

SO ORDERED.

Pakistan Intl Airlines v. Ople

(1990)

FACTS: 2 Filipino stewardess-employees of PIA filed a case for illegal dismissal against their employer in DOLE. PIA ’s defense is that under the contract of employment, the parties agreed that the EE-ER relationship shall be governed by the contract (which provided that Pakistan law shall apply) and not the Labor Code.

HELD: Public Interest standard was applied. Pakistan law cannot be invoked to prevent the application of Phil labor laws and regulations to the subject mat -ter of the case. The ER-EE relationship is much affected with public interest, such that otherwise applicable Phil laws and regulations cannot be rendered il -lusory by the parties agreeing upon some other law to govern their relationship.

Also, PIA did not undertake to plead and prove the contents of the Pakistan law on the matter; it must therefore be presumed to be the same as applicable provisions of Phil law.

Criticisms: courts using public policy exception can disregard the applicable law reached and replace it with forum law to arrive at its desired result without having to provide the rigorous legal analysis required to explain the shift. Courts engage in “intolerable affectation of superior virtue”.

PAKISTAN INTERNATIONAL AIRLINES (PIA) CORPORATION vs HON. BLAS F. OPLE G.R. No. 61594 September 28, 1990

FACTS: On 2 December 1978, petitioner Pakistan International Airlines Corporation (PIA), a foreign corporation licensed to do business in the Philippines, executed in Manila 2 separate contracts of employment, one with private respondent Farrales and the other with private respondent Mamasig. 1 The con-tracts, which became effective on 9 January 1979, provided in pertinent portion as follows:

5. DURATION OF EMPLOYMENT AND PENALTY

This agreement is for a period of 3 years, but can be extended by the mutual consent of the parties.

xxx xxx xxx

6. TERMINATION

xxx xxx xxx

Notwithstanding anything to contrary as herein provided, PIA reserves the right to terminate this agreement at any time by giving the EMPLOYEE notice in writing in advance one month before the intended termination or in lieu thereof, by paying the EMPLOYEE wages equivalent to one month’s salary.

xxx xxx xxx

10. APPLICABLE LAW:

This agreement shall be construed and governed under and by the laws of Pakistan, and only the Courts of Karachi, Pakistan shall have the jurisdiction to consider any matter arising out of or under this agreement.

Farrales & Mamasig (employees) were hired as flight attendants after undergoing training. Base station was in Manila and flying assignments to different parts of the Middle East and Europe.

roughly 1 year and 4 months prior to the expiration of the contracts of employment, PIA through Mr. Oscar Benares, counsel for and official of the local branch of PIA, sent separate letters, informing them that they will be terminated effective September 1, 1980.

Farrales and Mamasig jointly instituted a complaint, for illegal dismissal and non-payment of company benefits and bonuses, against PIA with the then Min-istry of Labor and Employment (MOLE).

PIA’s Contention: The PIA submitted its position paper, but no evidence, and there claimed that both private respondents were habitual absentees; that

both were in the habit of bringing in from abroad sizeable quantities of “personal effects”; and that PIA personnel at the Manila International Airport had been discreetly warned by customs officials to advise private respondents to discontinue that practice. PIA further claimed that the services of both private respondents were terminated pursuant to the provisions of the employment contract.

Favorable decision for the respondents. The Order stated that private respondents had attained the status of regular employees after they had rendered more than a year of continued service; that the stipulation limiting the period of the employment contract to 3 years was null and void as violative of the pro -visions of the Labor Code and its implementing rules and regulations on regular and casual employment; and that the dismissal, having been carried out without the requisite clearance from the MOLE, was illegal and entitled private respondents to reinstatement with full backwages.

Decision sustained on appeal. Hence, this petition for certiorari

ISSUE: (Relative to the subject) Which law should govern over the case? Which court has jurisdiction?

HELD: Philippine Law and Philippine courts

Petitioner PIA cannot take refuge in paragraph 10 of its employment agreement which specifies, firstly, the law of Pakistan as the applicable law of the agreement and, secondly, lays the venue for settlement of any dispute arising out of or in connection with the agreement “only [in] courts of Karachi Pak-

istan”.

We have already pointed out that the relationship is much affected with public interest and that the otherwise applicable Philippine laws and regulations cannot be rendered illusory by the parties agreeing upon some other law to govern their relationship.

the contract was not only executed in the Philippines, it was also performed here, at least partially; private respondents are Philippine citizens and respon-dents, while petitioner, although a foreign corporation, is licensed to do business (and actually doing business) and hence resident in the Philippines; lastly, private respondents were based in the Philippines in between their assigned flights to the Middle East and Europe. All the above contacts point to the Philippine courts and administrative agencies as a proper forum for the resolution of contractual disputes between the parties.

Under these circumstances, paragraph 10 of the employment agreement cannot be given effect so as to oust Philippine agencies and courts of the jurisdic-tion vested upon them by Philippine law. Finally, and in any event, the petitioner PIA did not undertake to plead and prove the contents of Pakistan law on the matter; it must therefore be presumed that the applicable provisions of the law of Pakistan are the same as the applicable provisions of Philippine law.

[DOCTRINE OF PROCESSUAL PRESUMPTION, eh?]

NOTES:bAnother Issue: petitioner PIA invokes paragraphs 5 and 6 of its contract of employment with private respondents Farrales and Mamasig, arguing that its relationship with them was governed by the provisions of its contract rather than by the general provisions of the Labor Code.

A contract freely entered into should, of course, be respected, as PIA argues, since a contract is the law between the parties. The principle of party auton -omy in contracts is not, however, an absolute principle. The rule in Article 1306, of our Civil Code is that the contracting parties may establish such stipula -tions as they may deem convenient, “provided they are not contrary to law, morals, good customs, public order or public policy.” Thus, counter-balancing the principle of autonomy of contracting parties is the equally general rule that provisions of applicable law, especially provisions relating to matters affected with public policy, are deemed written into the contract. Put a little differently, the governing principle is that parties may not contract away applicable provi -sions of law especially peremptory provisions dealing with matters heavily impressed with public interest. The law relating to labor and employment is clearly such an area and parties are not at liberty to insulate themselves and their relationships from the impact of labor laws and regulations by simply con -tracting with each other. It is thus necessary to appraise the contractual provisions invoked by petitioner PIA in terms of their consistency with applicable Philippine law and regulations.

Aznar vs. Garcia

(1963)

FACTS: The will of Edward Christensen, a domiciliary of the Philippines, was admitted to probate, and a project of partition was proposed. Edward’s illegiti-mate child opposed the project of partition on the ground that the distribution of the estate should be governed by Philippine law. The lower court found that Edward was a US citizen; hence the successional rights and intrinsic validity of the will should be governed by California.

HELD: Philippine law should apply. Art. 16 of the Phil. Civil Code provide that the national law of the decedent governs the validity of his testamentary dis-positions. Such national law means the law on conflict of laws of the California code, which authorizes the reference or return of the question to the law of the testator’s domicile. The conflict of laws rule in California precisely refers back the case, when a decedent is not domiciled in California, to the law of his domicile (the Philippines in the CAB). The Phil. court must apply its own law as directed in the conflict of laws rule of the state of the decedent.

3) by desistance or mutual disclaimer of JD – the same result as the acceptance of the renvoi doctrine but the process used by the forum court is to desist applying the foreign law.

4) “foreign court” theory – the forum court would assume the same position the foreign court would take were it litigated in the foreign state

Disadvantage to renvoi: if both courts follow the same theory, there would be no end to the case since the courts would be referring it back to each other. It gives rise to situations that have been invariably described as resembling “revolving doors,” a “game of lawn tennis,” a “logical cabinet of mirrors” or a “cir-culus inextricabilis.”

AZNAR vs. GARCIA G.R. No. L-16749 January 31, 1963

FACTS: EDWARD Christensen died testate. The estate was distributed by Executioner Aznar according to the will, which provides that: Php 3,600 be given to HELEN Christensen as her legacy, and the rest of his estate to his daughter LUCY Christensen, as pronounced by CFI Davao.

Opposition to the approval of the project of partition was filed by Helen, insofar as it deprives her of her legitime as an acknowledged natural child, she hav -ing been declared by Us an acknowledged natural child of the deceased Edward in an earlier case.

As to his citizenship, we find that the citizenship that he acquired in California when he resided in Sacramento from 1904 to 1913, was never lost by his stay in the Philippines, and the deceased appears to have considered himself as a citizen of California by the fact that when he executed his will he declared that he was a citizen of that State; so that he appears never to have intended to abandon his California citizenship by acquiring another. But at the time of his death, he was domiciled in the Philippines.

ISSUE: what law on succession should apply, the Philippine law or the California law?

HELD: WHEREFORE, the decision appealed from is hereby reversed and the case returned to the lower court with instructions that the partition be made as the Philippine law on succession provides.

The law that governs the validity of his testamentary dispositions is defined in Article 16 of the Civil Code of the Philippines, which is as follows:

ART. 16. Real property as well as personal property is subject to the law of the country where it is situated.

However, intestate and testamentary successions, both with respect to the order of succession and to the amount of successional rights and to the intrinsic validity of testamentary provisions, shall be regulated by the national law of the person whose succession is under consideration, whatever may be the na-ture of the property and regardless of the country where said property may be found.

The application of this article in the case at bar requires the determination of the meaning of the term “national law” is used therein.

The next question is: What is the law in California governing the disposition of personal property?

The decision of CFI Davao, sustains the contention of the executor-appellee that under the California Probate Code, a testator may dispose of his property by will in the form and manner he desires. But HELEN invokes the provisions of Article 946 of the Civil Code of California, which is as follows:

If there is no law to the contrary, in the place where personal property is situated, it is deemed to follow the person of its owner, and is governed by the law of his domicile.

It is argued on executor’s behalf that as the deceased Christensen was a citizen of the State of California, the internal law thereof, which is that given in the

Kaufman case, should govern the determination of the validity of the testamentary provisions of Christensen’s will, such law being in force in the State of California of which Christensen was a citizen. Appellant, on the other hand, insists that Article 946 should be applicable, and in accordance therewith and following the doctrine of the renvoi, the question of the validity of the testamentary provision in question should be referred back to the law of the decedent ’s domicile, which is the Philippines.

We note that Article 946 of the California Civil Code is its conflict of laws rule, while the rule applied in In re Kaufman, its internal law. If the law on succ es -sion and the conflict of laws rules of California are to be enforced jointly, each in its own intended and appropriate sphere, the principle cited In re Kaufman should apply to citizens living in the State, but Article 946 should apply to such of its citizens as are not domiciled in California but in other jurisdictions. The rule laid down of resorting to the law of the domicile in the determination of matters with foreign element involved is in accord with the general principle of American law that the domiciliary law should govern in most matters or rights which follow the person of the owner.

Appellees argue that what Article 16 of the Civil Code of the Philippines pointed out as the national law is the internal law of California. But as above ex -plained the laws of California have prescribed two sets of laws for its citizens, one for residents therein and another for those domiciled in other jurisdic -tions.

It is argued on appellees’ (Aznar and LUCY) behalf that the clause “if there is no law to the contrary in the place where the property is situated” in Sec. 946 of the California Civil Code refers to Article 16 of the Civil Code of the Philippines and that the law to the contrary in the Philippines is the provision in said Article 16 that the national law of the deceased should govern. This contention cannot be sustained.

As explained in the various authorities cited above, the national law mentioned in Article 16 of our Civil Code is the law on conflict of laws in the California Civil Code, i.e., Article 946, which authorizes the reference or return of the question to the law of the testator ’s domicile. The conflict of laws rule in Califor-nia, Article 946, Civil Code, precisely refers back the case, when a decedent is not domiciled in California, to the law of his domicile, the Philippines in the case at bar. The court of the domicile can not and should not refer the case back to California; such action would leave the issue incapable of determination because the case will then be like a football, tossed back and forth between the two states, between the country of which the decedent was a citizen and the country of his domicile. The Philippine court must apply its own law as directed in the conflict of laws rule of the state of the decedent, if the question has to be decided, especially as the application of the internal law of California provides no legitime for children while the Philippine law, Arts. 887(4) and 894, Civil Code of the Philippines, makes natural children legally acknowledged forced heirs of the parent recognizing them.

We therefore find that as the domicile of the deceased Edward, a citizen of California, is the Philippines, the validity of the provisions of his will depriving his acknowledged natural child, the appellant HELEN, should be governed by the Philippine Law, the domicile, pursuant to Art. 946 of the Civil Code of Califor-nia, not by the internal law of California..

NOTES: There is no single American law governing the validity of testamentary provisions in the United States, each state of the Union having its own pri -vate law applicable to its citizens only and in force only within the state. The “national law” indicated in Article 16 of the Civil Code above quoted can not, therefore, possibly mean or apply to any general American law. So it can refer to no other than the private law of the State of California.

Pfau vs. Trent Aluminum Co.

(1970)

FACTS: Trent (a New Jersey domiciliary) agreed to drive Pfau (a Connecticut domiciliary) to Missouri. While in Iowa, they had a vehicular accident causing injuries to Pfau. Pfau filed suit in New Jersey against Trent Aluminum Co. (registered owner of the car) for the damages he sustained while a passenger in Trent’s car. The defense of Trent Aluminum was that Iowa law is applicable, which provides that the host-driver is not liable to his passenger-guest for ordi -nary negligence.

HELD: Connecticut and New Jersey law both allow passenger-guest recovery. It appears that Connecticut’s substantive law allowing a guest to recover form his host’s ordinary negligence would give it a significant interest in having that law applied to this case. Since Iowa has no interest in this litigation, and since the substantive laws of Connecticut and New Jersey are the same, this case presents a false conflict and the Connecticut plaintiff should have the right to maintain an action for ordinary negligence in New Jersey.

PFAU V. TRENT ALUMINUM CO. SUPREME COURT OF NEW JERSEY 263 A.2D 129 (1970)

FACTS: Steven Pfau (plaintiff) and Bruce Trent (defendant) attended the same college in Iowa. Pfau was a domiciliary of Connecticut, Trent of New Jersey. While on campus, Trent operated a vehicle owned by his father’s company, Trent Aluminum Co. (defendant), a New Jersey corporation. Pfau was a pas-

senger in Trent’s vehicle when Trent got into an accident in Iowa. Pfau was injured. He sued Trent and Trent Aluminum in a New Jersey court for negli-

gence. Defendants asserted, as a defense, application of Iowa’s guest statute, which precluded a guest-passenger from suing the host-driver for ordinary negligence. Neither Connecticut nor New Jersey law contained a guest statute. In favor of Pfau, the trial court struck the guest statute defense. An appel -late court reversed. Pfau appealed to the Supreme Court of New Jersey. Defendants argued that if Connecticut law rather than Iowa law should apply, the court should then also apply Connecticut’s conflict-of-law rules, which designated the lex loci delicti—Iowa in this case—as the governing law for torts.

The opinion of the court was delivered by FOLEY, J.A.D.

With our leave defendants appeal from an interlocutory order of the Law Division by which the fourth separate defense set forth in defendants' answer was stricken. The precise question presented is whether, under the "grouping of contacts" or "center of gravity" approach adopted in this State by Mellk v. Sarahson, 49 N.J. 226 (1967), the law of Iowa which precludes an action, based upon ordinary negligence, by a guest in an automobile against his host, applies, or whether the law of New Jersey, which permits such a recovery, is applicable.

Plaintiff Steven Pfau was injured while riding as a guest passenger in a car driven by defendant Bruce Trent and owned by defendant Trent Aluminum Company. The accident occurred in Iowa on April 22, 1966 when the Trent car, while rounding a curve, crossed over the center line of a roadway and col-lided with a vehicle proceeding in the opposite direction.

Plaintiff, a domiciliary of Connecticut, was a student at Parsons College in Fairfield, Iowa. Defendant Bruce Trent, a New Jersey domiciliary, was also a stu -dent at that college. Both lived in college dormitories in Iowa. Trent Aluminum *326 Company is a New Jersey corporation which is owned by Bruce Trent's father. The occupants of the oncoming car, Joseph Davis, his wife and child, were Iowa residents and domiciliaries, and they all were seriously injured in the accident.

It was conceded on the oral argument that the Davis claims have been settled by defendants' insurance carrier, a New Jersey corporation, and that such claimants have no further interest in the matter. The vehicle driven by Trent was licensed and insured in New Jersey.

Pfau and Trent first met as students at Parsons College. Several days prior to April 22, 1966 Trent agreed to take Pfau with him on a trip to Columbia, Mis -souri. The boys were to stay over in Missouri and return to college in Iowa at the conclusion of the weekend. Pfau and Trent left Fairfield, Iowa, in the early evening of April 22, 1966. The accident occurred about one hour later.

The traditional rule for determining choice of law in tort cases has been lex loci delicti, the law of the place where the wrong occurred. Mellk v. Sarahson, supra, at page 228. However, in that case the court went on to say that recently there has been recognition by courts and commentators that the mechani -cal application of lex loci delicti to all choice-of-law problems may work unjust results in a particular case.

Following Babcock v. Jackson, 12 N.Y.2d 473, 240 N.Y.S.2d 743, 191 N.E.2d 279, 95 A.L.R.2d 1 (Ct. App. 1963), the Mellk court adopted the grouping of contacts or center of gravity principle and concluded that New Jersey had the paramount interest in fixing the rights and liabilities arising from the host-guest relationship and that the foreign state in which the accident occurred had no real interest in having its guest statute applied, pointing out, among other things, that both plaintiff and defendant were residents of New Jersey and that the host-guest relationship was formed and was to end here.

In Babcock the court, in refusing to apply an Ontario guest statute and holding that the concern of New York was unquestionably *327 the greater and more direct, the interest of Ontario being at best minimal, stated:

"The present action involves injuries sustained by a New York guest as the result of the negligence of a New York host in the operation of an automobile, garaged, licensed and undoubtedly insured in New York, in the course of a week-end journey which began and was to end there. In sharp contrast, On -tario's sole relationship with the occurrence is a purely adventitious circumstance that the accident occurred there." (240 N.Y.S.2d, at p. 750, 191 N.E.2d, at p. 284).

In Maffatone v. Woodson, 99 N.J. Super. 559 (App. Div. 1968), certification denied 51 N.J. 577 (1968), we upheld an instruction to the jury that the law of New York applied with respect to the liability and responsibility for a death and injuries to persons or property resulting from negligence in the use or opera-tion of such vehicle in the business of such owner, or otherwise, by using or operating the same with permission, express or implied, of such owner. See Vehicle and Traffic Law, McKinney's Consol. Laws, c. 71, § 388. In that case, a wrongful death action, decedent was killed in an automobile accident in New Jersey when the car driven by her husband and owned by his father was in collision with an automobile operated by a New Jersey resident. Citing Babcock and Mellk, supra, we held that in the circumstances, which included in addition to the facts stated, that the only purpose of a trip from New York to New Jer-sey was to pay respects to a deceased member of the family, after which, presumably, the car in which deceased was riding would return to New York, the substantive law of that state was controlling. It was our view that there were no New Jersey residents whose interest would be protected by the law of the forum state, whereas New York had an interest in the adequate compensation for the wrongful death of a resident.

Plaintiff cites and places considerable reliance upon Mullane v. Stavola, 101 N.J. Super. 184 (Law Div. 1968). With *328 out expressing any view as to the holding in Mullane, we conclude that it is readily distinguishable in the important respect that in that case both plaintiff's decedent and defendant were domi -ciliaries of New Jersey. There, the court said:

"As I view it, * * * New Jersey law should be applied in the instant case. Even though Florida's `contacts' may be quantitatively greater than New Jersey's they are qualitatively less significant. The salient consideration is that all the parties at the time of the accident were domiciled in New Jersey. Their Florida residences were temporary. New Jersey's interest and concern in the parties as domiciliaries was greater than Florida's interest in them as temporary resi-dents, hence New Jersey's public policy requiring drivers to exercise greater care for passengers than Florida does, should control." (101 N.J. Super., at p. 189; emphasis added)

In the present case the basis for the trial court determination was that as between Iowa and New Jersey, the latter had the more significant contacts with the parties and the greater governmental interest in the matter. We cannot agree.

The only alleged contacts of New Jersey were that defendants were domiciliaries of this state and their vehicle was licensed in New Jersey and insured by a New Jersey corporation. As we have noted, the ill-fated trip commenced in Iowa and was designed to terminate in that state, and the accident occurred there. Moreover, the parties to the instant litigation were temporary residents of that state and the injured persons in the other car were residents of that state. Furthermore, the host-guest relationship came into being in Iowa. Lastly, Pfau was a domiciliary of Connecticut which is a lex loci delicti jurisdiction.

It is our judgment that upon the facts of the instant case it clearly appears that applying the "center of gravity" or "most significant contacts" test Iowa had the more significant contacts with the parties to this litigation.

Therefore, for the reasons stated, we conclude that the guest statute of Iowa was properly pleaded by defendants. Accordingly, the interlocutory order of the Law Division is *329 set aside with the direction that the fourth separate defense of defendants' answer be reinstated. Reversed.

Bellis vs. Bellis

(1968)

FACTS: The probate of the will of Texas citizen and domiciliary Amos Bellis was opposed by his 3 illegitimate children in the Philippines for depriving them of their compulsory legitime. However, the trial court ruled that under Art. 16 of the Phil. Civil Code, the national law of the decedent is to be applied in tes-tamentary succession. The law of Texas did not provide for legitimes.

HELD: Texas law should apply. The decedent was both a national and a domiciliary of Texas, so that even assuming Texas has a conflict of law rule pro -viding that the law of the domicile should govern, the rule would not result in a reference back (renvoi) to Philippine law, but would still refer to Texas law.

Bellis vs. Bellis

G.R. No. L-23678.

June 6, 1967

Facts:

Amos Bellis was a citizen and a resident of Texas when he died. Before his death, he executed a will in the Philippines wherein he directed that his net estate would be distributed in the following manner: (a) $240,000.00 to his first wife, Mary E. Mallen; (b) P120,000.00 to his three illegitimate chil-dren and the (c) the remainder shall go to his seven surviving children by his first and second wives. Among those surviving children are Maria Cristina Bel-lis and Miriam Palma Bellis, the oppositors. The People's Bank and Trust Company acted as executor of the will. Prior to the submission of final account, Maria Cristina Bellis and Miriam Palma Bellis filed their respective oppositions to the project of partition on the ground that they were deprived of their legit-imes as illegitimate children and compulsory heirs of the deceased. They contend that although Article 16 and Article 1039 of the Civil Code provide that it is the national law of the decedent which governs the following: (a) the order of succession; (b) the amount of successional rights; (c) the intrinsic validity of the provisions of the will; and (d) the capacity to succeed , Article 17 par. 3 of the Civil Code, stating that —

"Prohibitive laws concerning persons, their acts or property, and those which have for their object public order, public policy and good customs shall not be rendered ineffective by laws, or judgments promulgated, or by determinations or conventions agreed upon in a foreign country."

prevails as the exception. Furthermore, they argue that the decedent, in executing two wills, intended one to govern his Texas estate and the other his Philippine estate.

Issue:

Whether or not Art. 17 of the Civil Code prevails as an exception to Art. 16 and Art. 1039

Ruling:

(1) No, Art. 16 par. 2 of the Civil Code which states that “intestate and testamentary successions, both with respect to the order of suc-cession and to the amount of successional rights and to the intrinsic validity of testamentary provisions, shall be regulated by the national law of the person whose succession is under consideration, whatever may be the nature of the property and regard -less of the country wherein said property may be found" is a specific provision in itself which is applied in testate and intestate succes-sions. As further indication of this legislative intent, Congress added a new provision, under Art. 1039, which decrees that capacity to suc-ceed is to be governed by the national law of the decedent.

Accordingly, since the parties admit that the decedent, Amos G. Bellis, was a citizen of the State of Texas, U.S.A., and that under the laws of Texas, there are no forced heirs or legitimates, the Philippine law on legitimes cannot be applied to the testacy of Amos G. Bellis.

(2) Moreover, even assuming that in executing a separate Philippine will, the decedent intended that Philippine law govern his Philippine es-tate, such intention is illegal and void for his national law cannot be ignored in regard to those matters that Article 16 of the Civil Code states should be governed by the national of the decedent.

G.R. No. 3474, Enriquez et al. v. Enriquez et al., 8 Phil. 565

The plaintiffs brought this action in the Court of First Instance of Manila on the 2d day of June, 1902, asking that a deed made by Antonio Enriquez on the 27th of March, 1883, conveying to the defendant Carmen de la Cavada certain real estate in the city of Manila, be annulled and set aside. Judgment was rendered in the court below to the effect that the plaintiffs were the owners of an undivided half of the said real estate, and that the defendant Carmen de la Cavada should pay to the plaintiffs upward of 1,300 pesos, as rents and profits thereof. Both parties moved for a new trial on the ground of the insufficiency of the evidence, by the plaintiffs in this court have neither assigned as errors the rulings made against them, by the lower court nor have they discussed any

such rulings in their brief. So much of the decision, therefore, as is adverse to the plaintiffs we can not consider, and the questions to be resolved are those presented by the appeal of the defendants.

The decision of the court below was based upon the following facts, deemed to be established by the evidence, namely, that Antonio Enriquez and Do ña Ciriaca Villanueva were legally married prior to the year 1860; that in 1861 the property in question was acquired by Antonio Enriquez; that it thereby be-came a part of the property belonging to the conjugal partnership; that Doña Ciriaca Villanueva died in 1882; that upon her death an undivided half of the

property passed to her heirs, the plaintiffs; that when, in 1883, Antonio Enriquez undertook to convey the entire property to the defendant Doña Carmen de la Cavada he, as matter of law, conveyed one half thereof, and that the other half remained and now is the property of the plaintiffs.

The correctness of this decision depends upon the question as to whether Antonio Enriquez and Doña Ciriaca Villanueva were legally married in 1861. The court below found and, the evidence sustains that finding, that a marriage ceremony was duly performed between these persons in 1865, but held that the fact that prior to 1861 they had lived together as husband and wife, had been recognized as such, and had had children who were baptized as the legiti-mate children of their lawful marriage was sufficient evidence to raise the presumption that they were at the time legally married.

A marriage ceremony having been duly celebrated between these persons in 1865, it is necessary, in order to show that they were legally married before that time, to prove that the same kind of a marriage ceremony had theretofore been celebrated. Although, as held by the Supreme Court of the United States, by the common law of England, a valid marriage might be contacted without the intervention of any ecclesiastical or civil functionary (Traverse vs. Rheinhardt, 27 Sup. Ct. Rep., 563, decided April 15, 1907), yet such was never the law in these Islands during the Spanish domination here. During the en-tire period of that domination no valid marriage could exist unless some ecclesiastical or civil functionary intervened in its celebration, and the intervention of civil functionaries was limited to the short time elapsing between the 8th day of December, 1889, when the Civil Code took effect here, and the 29th day of the same month, when the provisions of Title IV, Book I, of that code were suspended. During the time covered by the lives of Antonio Enriquez and Doña Ciriaca Villanueva no valid marriage between them could be contracted by their mere agreement to live together as husband and wife.

There is no proof in this case that a marriage, valid in accordance with the laws then in force in these Islands, was celebrated between these persons in 1865. In order to show that they were before that time husband and wife, it was necessary to prove that a marriage ceremony in which an ecclesiastical functionary intervened was duly celebrated. No proof of any such marriage was offered. As has been said, the fact that prior to 1865 they lived together as husband and wife and had children is not evidence in this case to show that they were married prior to that time. Nor is the fact that in the certificates of baptism of these children it is stated that they were the legitimate children of the lawful marriage of their parents.

The court below said: Loss of the record of the first marriage, or some like reason, might have made the second ceremony seem necessary and for that reason it was celebrated.

This consideration is to our minds entirely insufficient to explain the celebration of the second marriage. If the former marriage had taken place, it must have been celebrated before some priest or other officer of the Roman Catholic Church. The law required that a record of such marriages should be kept in the parish registry, and if such marriage in fact had been performed, it probably would have been easy to have obtained a certified copy of such record. No evi -dence was offered in this case of any attempt to obtain such record or that the records of the church were the ceremony had been performed had been destroyed. In fact, no proof whatever was offered in the case to show the celebration of such prior marriage, except the facts hereinbefore stated, that the parties lived together as husband and wife and had children who were baptized as aforesaid. We hold that this evidence is insufficient to prove in this case a prior marriage, where it appears that a marriage ceremony was duly performed between the parties at a later date; and we therefore hold that Antonio En -riquez and Doña Ciriaca Villanueva were not legally married prior to 1865, and that, therefore, when this property was acquired by Antonio in 1861 it did not become a part of the property belonging to the conjugal partnership, but on the contrary was a part of the capital which he brought to the marriage. Being a part of the capital brought to the marriage by the husband, upon the death of the wife — the husband surviving her — no interest whatever therein passed to her heirs.

The judgment of the court below, which rests solely upon the proposition that at the time of the death of Doña Ciriaca Villanueva one-half of this property passed to her heirs, can not, therefore, be sustained. That judgment is reversed, without costs to either in this court, judgment is entered acquitting the de-fendants of the complaint, with the costs of the first instance against the plaintiffs. So ordered.

Eugenio vs Velez 185 SCRA 45

FACTS: Vitaliana Vargas’ brothers and sisters unaware of the former’s death on August 28, 1988 filed a petition for Habeas Corpus on September 27, 1988 before the RTC of Misamis Oriental alleging that she was forcible taken from her residence sometime in 1987 and was confined by the herein petitioner, Tomas Eugenio in his palacial residence in Jasaan, Misamis Oriental. The court then issued a writ of habeas corpus but petitioner refused to surrender the Vitaliana’s body to the sheriff on the ground that a corpse cannot be subjected to habeas corpus proceedings. Vitaliana, 25 year old single, died of heart

failure due to toxemia of pregnancy in Eugenio’s residence. The court ordered that the body should be delivered to a funeral parlor for autopsy but Eugenio assailed the lack of jurisdiction of the court.

ISSUE: Whether or not the petitioner can claim custody of the deceased.

HELD: The court held that the custody of the dead body of Vitaliana was correctly awarded to the surviving brothers and sisters pursuant to Section 1103 of the Revised Administrative Code which provides:

“Persons charged with duty of burial- if the deceased was an unmarried man or woman or a child and left any kin; the duty of the burial shall devolve upon the nearest kin of the deceased.

Albeit, petitioner claims he is the spouse as contemplated under Art. 294 of the Civil Code, Philippine law does not recognize common law marriages where “a man and a woman not legally married who cohabit for many years as husband and wife, who represent themselves to the public as husband and wife,

and who are reputed to be husband and wife in the community where they live may be considered legally mauled in common law jurisdictions ”. In addition, it requires that the man and woman living together must not in any way be incapacitated to contract marriage. Whereas, the petitioner has a subsisting marriage with another woman, legal impediment that disqualified him from even legally marrying Vitaliana.

Djumantan vs. Domingo

Facts: Bernard Banez, the husband of Marina Cabael, went to Indonesia as a contract worker.

On April 3, 1974, he embraced and was converted to Islam. On May 17, 1974, he married petitioner in accordance with Islamic rites. He returned to the Phi-lippines in January 1979. On January 13, 1979, petitioner and her two children with Banez, arrived in Manila as the "guests" of Banez. The latter made it appear that he was just a friend of the family of petitioner and was merely repaying the hospitability extended to him during his stay in Indonesia. When pe -titioner and her two children arrived at the Ninoy Aquino International Airport on January 13, 1979, Banez, together with Marina Cabael, met them.As "guests," petitioner and her two children lived in the house of Banez. Petitioner and her children were admitted to the Philippines as temporary visitors un-der Section 9(a) of the Immigration Act of 1940. In 1981, Marina Cabael discovered the true relationship of her husband and petitioner. On March 25, 1982, the immigration status of petitioner was changed from temporary visitor to that of permanent resident under Section 13(a) of the same law. On April 14, 1982, petitioner was issued an alien certificate of registration.

Not accepting the set-back, Banez' eldest son, Leonardo, filed a letter complaint with the Ombudsman, who subsequently referred the letter to the CID. On the basis of the said letter, petitioner was detained at the CID detention cell. The CID issued an order revoking the status of permanent resident given to petitioner, the Board found the 2nd marriage irregular and not in accordance with the laws of the Phils. There was thus no basis for giving her the status of permanent residence, since she was an Indonesian citizen and her marriage with a Filipino Citizen was not valid.

Thus this petition for certiorari

Issue: Whether or not the courts may review deportation proceedings

Held : Yes. Section 1 of Article 8 says Judicial Power includes 1) settle actual controversies involving rights which are legally demandable and enforceable 2) determine whether or not there has been a grave abuse of discretion amounting to lack or excess of jurisdiction on the part of any branch or instrumenta -lity of the government.

We need not resolve the validity of petitioner's marriage to Banez, if under the law the CID can validly deport petitioner as an "undesirable alien" regardless of her marriage to a Filipino citizen. Generally, the right of the President to expel or deport aliens whose presence is deemed inimical to the public interest is as absolute and unqualified as the right to prohibit and prevent their entry into the country.

However, under clause 1 of Section 37(a) of the Immigration Act of 1940 an "alien who enters the Philippines after the effective date of this Act by means of false and misleading statements or without inspection and admission by the immigration authorities at a designated port of entry or at any place other than at a designated port of entry" is subject to deportation. The deportation of an alien under said clause of Section 37(a) has a prescriptive period and "shall not be effected ... unless the arrest in the deportation proceedings is made within five years after the cause for deportation arises". Tolling the prescriptive period from November 19, 1980, when Leonardo C. Banez informed the CID of the illegal entry of petitioner into the country, more than five years had elap-sed before the issuance of the order of her deportation on September 27, 1990.

Goitia vs. Campos-Rueda 35 Phil 252

FACTS: Luisa Goitia y de la Camara, petitioner, and Jose Campos y Rueda, respondent, were married on January 7, 1915 and had a residence at 115 Calle San Marcelino Manila. They stayed together for a month before petitioner returned to her parent ’s home. Goitia filed a complaint against respondent for support outside the conjugal home. It was alleged that respondent demanded her to perform unchaste and lascivious acts on his genital organs. Peti -tioner refused to perform such acts and demanded her husband other than the legal and valid cohabitation. Since Goitia kept on refusing, respondent mal -treated her by word and deed, inflicting injuries upon her lops, face and different body parts. The trial court ruled in favor of respondent and stated that Goi -tia could not compel her husband to support her except in the conjugal home unless it is by virtue of a judicial decree granting her separation or divorce from respondent. Goitia filed motion for review.

ISSUE: Whether or not Goitia can compel her husband to support her outside the conjugal home.

HELD: The obligation on the part of the husband to support his wife is created merely in the act of marriage. The law provides that the husband, who is obliged to support the wife, may fulfill the obligation either by paying her a fixed pension or by maintaining her in his own home at his option. However, this option given by law is not absolute. The law will not permit the husband to evade or terminate his obligation to support his wife if the wife is driven away from the conjugal home because of his wrongful acts. In the case at bar, the wife was forced to leave the conjugal abode because of the lewd designs and physical assault of the husband, she can therefore claim support from the husband for separate maintenance even outside the conjugal home.

ARROYO v VASQUEZ (1921)

FACTS: Plaintiff Mariano and defendant Dolores were married in 1910, and lived in Iloilo City. They lived together with a few short intervals of separation. On July 4, 1920, defendant Dolores went away from their common home and decided to live separately from plaintiff. She claimed that she was compelled to leave on the basis of cruel treatment on the part of her husband. She in turn prayed for a decree of separation, a liquidation of their conjugal partnership, and an allowance for counsel fees and permanent separate maintenance.

CFI ruled in favor of the defendant and she was granted alimony amounting to P400, also other fees

Plaintiff then asked for a restitution of conjugal rights, and a permanent mandatory injunction requiring the defendant to return to the conjugal home and live with him as his wife.

ISSUES:

1. WON defendant had sufficient cause for leaving the conjugal home

2. WON plaintiff may be granted the restitution of conjugal rights or absolute order or permanent mandatory injunction

HELD:

1. The wife had sufficient cause for leaving the conjugal home. Cruelty done by plaintiff to defendant was greatly exaggerated. The wife was inflicted with a disposition of jealousy towards her husband in an aggravated degree. No sufficient cause was present.

Courts should move with caution in enforcing the duty to provide for the separate maintenance of the wife since this recognizes the de facto separation of the two parties. Continued cohabitation of the pair must be seen as impossible, and separation must be necessary, stemming from the fault of the husband. She is under obligation to return to the domicile.

“When people understand that they must live together…they learn to soften by mutual accommodation that yoke which they know they cannot shake off;

they become good husbands and wives…necessity is a powerful master in teaching the duties which it imposes…” (Evans v. Evans)

2. On granting the restitution of conjugal rights. It is not within the province of the courts to compel one of the spouses to cohabit with, and render conjugal rights to, the other. In the case of property rights, such an action may be maintained. Said order, at best, would have no other purpose than to compel the spouses to live together. Other countries, such as England and Scotland have done this with much criticism.

Plaintiff is entitled to a judicial declaration that the defendant absented herself without sufficient cause and it is her duty to return. She is also not entitled to support.

Tenchavez vs. Escaño

(1965)

FACTS: Vicenta and Pastor were married without the knowledge of her parents. Vicenta went to the US to obtain a divorce, which was granted by the Nevada Court; she married an American and subsequently acquired American citizenship. Pastor sued Vicenta for legal separation and damages. Vi-centa’s defense is that there was a valid divorce issued by the Nevada court.

HELD: The divorce decree obtained in the US is not valid, because at the time it was issued, Vicenta, like Pastor, was still a Filipino citizen. She was then subject to Philippine law. Philippine law cannot recognize a foreign decree of absolute divorce between Filipino citizens, for this would violate declared pub-lic policy.

Tenchavez vs Escano 15 SCRA 355

FACTS: 27 years old Vicenta Escano who belong to a prominent Filipino Family of Spanish ancestry got married on Feburary 24, 1948 with Pastor Tencha-vez, 32 years old engineer, and ex-army officer before Catholic chaplain Lt. Moises Lavares. The marriage was a culmination of the love affair of the couple and was duly registered in the local civil registry. A certain Pacita Noel came to be their match-maker and go-between who had an amorous rela -tionship with Tenchavez as written by a San Carlos college student where she and Vicenta are studying. Vicenta and Pastor are supposed to renew their vows/ marriage in a church as suggested by Vicenta’s parents. However after translating the said letter to Vicenta’s dad , he disagreed for a new marriage. Vicenta continued leaving with her parents in Cebu while Pastor went back to work in Manila.

Vicenta applied for a passport indicating that she was single and when it was approved she left for the United States and filed a complaint for divorce against Pastor which was later on approved and issued by the Second Judicial Court of the State of Nevada. She then sought for the annulment of her marriage to the Archbishop of Cebu. Vicenta married Russell Leo Moran, an American, in Nevada and has begotten children. She acquired citizenship on August 8, 1958. Petitioner filed a complaint against Vicenta and her parents whom he alleged to have dissuaded Vicenta from joining her husband.

ISSUE: Whether the divorce sought by Vicenta Escano is valid and binding upon courts of the Philippines.

HELD: Civil Code of the Philippines does not admit divorce. Philippine courts cannot give recognition on foreign decrees of absolute divorce between Filipi -no citizens because it would be a violation of the Civil Code. Such grant would arise to discrimination in favor of rich citizens who can afford divorce in fo -reign countries. The adulterous relationship of Escano with her American husband is enough grounds for the legal separation prayed by Tenchavez. In the eyes of Philippine laws, Tenchavez and Escano are still married. A foreign divorce between Filipinos sought and decreed is not entitled to recognition nei -ther is the marriage of the divorcee entitled to validity in the Philippines. Thus, the desertion and securing of an invalid divorce decree by one spouse entit -led the other for damages.

WHEREFORE, the decision under appeal is hereby modified as follows;

(1) Adjudging plaintiff-appellant Pastor Tenchavez entitled to a decree of legal separation from defendant Vicenta F. Escaño;

(2) Sentencing defendant-appellee Vicenta Escaño to pay plaintiff-appellant Tenchavez the amount of P25,000 for damages and attorneys' fees;

(3) Sentencing appellant Pastor Tenchavez to pay the appellee, Mamerto Escaño and the estate of his wife, the deceased Mena Escaño, P5,000 by way of damages and attorneys' fees.

Van Dorn vs. Romillo

(1985)

FACTS: Van Dorn, a Filipina, married American Upton. Ten years later they were divorced in the US; subsequently, van Dorn remarried. Upton filed an ac-tion against Van Dorn in the Philippines, asking for an accounting of certain alleged conjugal properties.

HELD: The divorce decree is valid and binding upon Upton. Even if divorce is not valid in the Philippines for being contrary to public policy, only Philippine nationals are covered by the policy against absolute divorces. Aliens may obtain divorces abroad, which may be recognized in the Philippines provided valid according to their national law.

Van Dorn v. Romillo

G.R. No. L-68470 October 8, 1985

Facts: Petitioner Alice Reyes Van Dorn is a citizen of the Philippines while private respondent, Richard Upton is a citizen of the United States; that they were married in Hongkong in 1972; that, after the marriage, they established their residence in the Philippines; that they begot two children born on April 4, 1973 and December 18, 1975, respectively; that the parties were divorced in Nevada, United States, in 1982; and that petitioner has re-married also in Nevada, this time to Theodore Van Dorn.

On June 8, 1983, private respondent filed suit against petitioner before the RTC of Pasay City, stating that petitioner's business in Ermita, (the Galleon Shop), is conjugal property of the parties, and asking that petitioner be ordered to render an accounting of that business, and that private respondent be declared with right to manage the conjugal property. Petitioner moved to dismiss the case on the ground that the cause of action is barred by previous judgment in the divorce proceedings before the Nevada Court wherein respondent had acknowledged that he and petitioner had "no community property" as of June 11, 1982. The Court below denied the Motion to Dismiss in the mentioned case on the ground that the property involved is located in the Philippines so that the Divorce Decree has no bearing in the case. The denial is now the subject of this certiorari proceeding.

Issue: Whether the divorce decree obtained from the Nevada Court should be given effect in the Philippines.

Held: Yes.

Petitioner contends that respondent is estopped from laying claim on the alleged conjugal property because of the representation he made in the divorce proceedings before the American Court that they had no community of property; that the Galleon Shop was not established through conjugal funds, and that respondent's claim is barred by prior judgment.

For his part, respondent avers that the Divorce Decree issued by the Nevada Court cannot prevail over the prohibitive laws of the Philippines and its declared national policy; that the acts and declaration of a foreign Court cannot, especially if the same is contrary to public policy, divest Philippine Courts of jurisdiction to entertain matters within its jurisdiction.

The Nevada District Court, which decreed the divorce, had obtained jurisdiction over petitioner who appeared in person before the Court during the trial of the case. It also obtained jurisdiction over private respondent who, giving his address as No. 381 Bush Street, San Francisco, California, authorized his attorneys in the divorce case, Karp & Gradt Ltd., to agree to the divorce on the ground of incompatibility in the understanding that there were neither community property nor community obligations.

There can be no question as to the validity of that Nevada divorce in any of the States of the United States. The decree is binding on private respondent as an American citizen. For instance, private respondent cannot sue petitioner, as her husband, in any State of the Union. What he is contending in this case is that the divorce is not valid and binding in this jurisdiction, the same being contrary to local law and public policy.

It is true that owing to the nationality principle embodied in Article 15 of the Civil Code, only Philippine nationals are covered by the policy against absolute divorces the same being considered contrary to our concept of public police and morality. However, aliens may obtain divorces abroad, which may be recognized in the Philippines, provided they are valid according to their national law. In this case, the divorce in Nevada released private respondent from the marriage from the standards of American law, under which divorce dissolves the marriage. As stated by the Federal Supreme Court of the United States in Atherton vs. Atherton, 45 L. Ed. 794, 799:

The purpose and effect of a decree of divorce from the bond of matrimony by a court of competent jurisdiction are to change the existing status or domestic relation of husband and wife, and to free them both from the bond. The marriage tie when thus severed as to one party, ceases to bind either. A husband without a wife, or a wife without a husband, is unknown to the law. When the law provides, in the nature of a penalty. that the guilty party shall not marry again, that party, as well as the other, is still absolutely freed from the bond of the former marriage.

Thus, pursuant to his national law, private respondent is no longer the husband of petitioner. He would have no standing to sue in the case below as petitioner's husband entitled to exercise control over conjugal assets. As he is bound by the Decision of his own country's Court, which validly exercised jurisdiction over him, and whose decision he does not repudiate, he is estopped by his own representation before said Court from asserting his right over the alleged conjugal property.

To maintain, as private respondent does, that, under our laws, petitioner has to be considered still married to private respondent and still subject to a wife's obligations under Article 109, et. seq. of the Civil Code cannot be just. Petitioner should not be obliged to live together with, observe respect and fidelity, and render support to private respondent. The latter should not continue to be one of her heirs with possible rights to conjugal property. She should not be discriminated against in her own country if the ends of justice are to be served.

Pilapil vs. Ibay Somera

(1989)

FACTS: Pilapil and Geiling were married in Germany, but were later divorced. A few months later, Geiling filed a complaint for adultery which was dis -missed; it was refiled by the fiscal.

HELD: Geiling has no legal standing to commence the adultery case because the person who initiates the adultery case must be an offended spouse, meaning he must still be married to the accused spouse at the time of the filing of the complaint. Because of the divorce decree, Geiling is no longer the husband of Pilapil; hence he had no more legal standing to commence the adultery case (no longer an offended “spouse”).

PILAPIL vs. HON IBAY-SOMERA, VICTOR AND GEILING et al G.R. No. 80116 June 30, 1989

FACTS: Petitioner Imelda Pilapil, a Filipino citizen, and private respondent Erich Geiling, a German national, were married in Germany. After about three and a half years of marriage, such connubial disharmony eventuated in Geiling initiating a divorce proceeding against Pilapil in Germany. The Local Court, Federal Republic of Germany, promulgated a decree of divorce on the ground of failure of marriage of the spouses.

More than five months after the issuance of the divorce decree, Geiling filed two complaints for adultery before the City Fiscal of Manila alleging in one that, while still married to said Geiling, Pilapil “had an affair with a certain William Chia.” The Assistant Fiscal, after the corresponding investigation, recommen-ded the dismissal of the cases on the ground of insufficiency of evidence. However, upon review, the respondent city fiscal Victor approved a resolution di -recting the filing of 2 complaint for adultery against the petitioner. The case entitled “PP Philippines vs. Pilapil and Chia” was assigned to the court presided by the respondent judge Ibay-Somera.

A motion to quash was filed in the same case which was denied by the respondent. Pilapil filed this special civil action for certiorari and prohibition, with a prayer for a TRO, seeking the annulment of the order of the lower court denying her motion to quash.

As cogently argued by Pilapil, Article 344 of the RPC thus presupposes that the marital relationship is still subsisting at the time of the institution of the cri -minal action for adultery.

ISSUE: Did Geiling have legal capacity at the time of the filing of the complaint for adultery, considering that it was done after obtaining a divorce decree?

HELD: WHEREFORE, the questioned order denying petitioner’s MTQ is SET ASIDE and another one entered DISMISSING the complaint … for lack of ju-

risdiction. The TRO issued in this case … is hereby made permanent.

NO. Under Article 344 of the RPC, the crime of adultery cannot be prosecuted except upon a sworn written complaint filed by the offended spouse. It has long since been established, with unwavering consistency, that compliance with this rule is a jurisdictional, and not merely a formal, requirement.

Corollary to such exclusive grant of power to the offended spouse to institute the action, it necessarily follows that such initiator must have the status, capa -city or legal representation to do so at the time of the filing of the criminal action. This is a logical consequence since the raison d ’etre of said provision of law would be absent where the supposed offended party had ceased to be the spouse of the alleged offender at the time of the filing of the criminal case.

Stated differently, the inquiry would be whether it is necessary in the commencement of a criminal action for adultery that the marital bonds between the complainant and the accused be unsevered and existing at the time of the institution of the action by the former against the latter.

In the present case, the fact that private respondent obtained a valid divorce in his country, the Federal Republic of Germany, is admitted. Said divorce and its legal effects may be recognized in the Philippines insofar as private respondent is concerned in view of the nationality principle in our civil law on the matter of status of persons Under the same considerations and rationale, private respondent, being no longer the husband of petitioner, had no legal stan -ding to commence the adultery case under the imposture that he was the offended spouse at the time he filed suit.

Quita vs. Court of Appeals

(1998)

FACTS: Quita and Padlan were married in the Philippines, but Quita filed for divorce in California which was granted. She remarried twice after the divorce. Upon Padlan’s death, Quita made claims upon his estate as the surviving spouse and heir of Padlan, alleging that since Padlan was a Filipino citizen, he remained married to her in spite of the divorce decree.

HELD: Quita’s right to inherit from Padlan depends on her citizenship at the time the divorce was decreed. If she was no longer a Filipino citizen at the time of their divorce, the divorce would be valid as to her and will be recognized in the Philippines, and she would lose her right to inherit.

✪ Tenchavez: 2 Filipinos, divorced abroad – not recognized in the Philippines (Art. 15 CC, nationality principle)

✪ Van Dorn: 1 Filipino, 1 foreigner, divorced abroad – recognized (Art. 15 CC, estoppel)

✪ Pilapil: 1 Filipino, 1 foreigner – recognized (nationality principle)

Quita vs Court of Appeals December 22, 1998

Fact of the Case: Fe D. Quita, the petitioner, and Arturo T. Padlan, both Filipinos, were married in the Philippines on May 18, 1941. They got divorce in San Francisco on July 23, 1954. Both of them remarried another person. Arturo remarried Bladina Dandan, the respondent herewith. They were blessed with six children.

On April 16, 1972, when Arturo died, the trial court was set to declared as to who will be the intestate heirs. The trial court invoking Tenchavez vs Escano case held that the divorce acquired by the petitioner is not recognized in our country.

Private respondent stressed that the citizenship of petitioner was relevant in the light of the ruling in Van Dorn v. Rommillo Jr that aliens who obtain divorce abroad are recognized in the Philippnes provided they are valid according to their national law. The petitioner herself answered that she was an American citizen since 1954. Through the hearing she also stated that Arturo was a Filipino at the time she obtained the divorce. Implying the she was no longer a Fi -lipino citizen. The Trial court disregarded the respondent’s statement. The net hereditary estate was ordered in favor the Fe D. Quita and Ruperto, the bro -ther of Arturo. Blandina and the Padlan children moved for reconsideration.

On February 15, 1988 partial reconsideration was granted declaring the Padlan children, with the exception of Alexis, entitled to one- half of the estate to the exclusion of Ruperto Padlan, and the other half to Fe Quita. Private respondent was not declared an heir for her marriage to Arturo was declared void since it was celebrated during the existence of his previous marriage to petitioner. Blandina and her children appeal to the Court of Appeals thatthe case was decided without a hearing in violation of the Rules of Court.

Notes!

Issue: (1) Whether or not Blandina’s marriage to Arturo void ab initio. (2) Whether or not Fe D. Quita be declared the primary beneficiary as surviving spouse of Arturo.

Held: No. The marriage of Blandina and Arturo is not void. The citizenship of Fe D. Quita at the time of their divorce is relevant to this case. The divorce is valid here since she was already an alien at the time she obtained divorce, and such is valid in their country ’s national law. Thus, Fe D. Quita is no longer recognized as a wife of Arturo. She cannot be the primary beneficiary or will be recognized as surviving spouse of Arturo.

Quita v. CAG.R. No. 124862 December 22, 1998

Facts: Fe D. Quita and Arturo T. Padlan, both Filipinos, were married in the Philippines on 18 May 1941. They were not however blessed with children. When their marriage became sour, Fe sued Arturo for divorce in San Francisco, California. She submitted in the divorce proceedings a private writing dated 19 July 1950 evidencing their agreement to live separately from each other and a settlement of their conjugal properties. On 23 July 1954 she obtained a final judgment of divorce. Three (3) weeks thereafter she married a certain Felix Tupaz in the same locality but their relationship also ended in a divorce. Still in the U.S.A., she married for the third time, to a certain Wernimont.

On 16 April 1972 Arturo died without will. On August 31, 1972 Lino Javier Inciong filed a petition with the QCRTC for issuance of letters of administration concerning the estate of Arturo in favor of the Philippine Trust Company. Respondent Blandina Dandan, claiming to be the surviving spouse of Arturo and Claro, Alexis, Ricardo, Emmanuel, Zenaida and Yolanda, all surnamed Padlan, named in the petition as surviving children of Arturo, opposed the petition and prayed for the appointment instead of Atty. Leonardo Cabasal, which was resolved in favor of the latter. Upon motion of the oppositors themselves, Atty. Cabasal was later replaced by Higino Castillon. On 30 April 1973 the oppositors (Blandina and the Padlan children) submitted certified photocopies of the 19 July 1950 private writing and the final judgment of divorce between petitioner and Arturo.

On 7 October 1987 petitioner moved for the immediate declaration of heirs of the decedent and the distribution of his estate. At the scheduled hearing on 23 October 1987, private respondent as well as the six (6) Padlan children and Ruperto failed to appear despite due notice.

The trial court invoking Tenchavez v. Escaño which held that "a foreign divorce between Filipino citizens sought and decreed after the effectivity of the present Civil Code (Rep. Act 386) was not entitled to recognition as valid in this jurisdiction," disregarded the divorce between petitioner and Arturo. Consequently, it expressed the view that their marriage subsisted until the death of Arturo in 1972. Neither did it consider valid their extrajudicial settlement of conjugal properties due to lack of judicial approval.

Private respondent Blandina was not declared an heir. Although it was stated in the aforementioned records of birth that she and Arturo were married on 22 April 1947, their marriage was clearly void since it was celebrated during the existence of his previous marriage to petitioner.

Issue: Whether petitioner’s marriage with the Arturo subsisted despite the divorce decree granted by the California court and subsequent marriages thereby entitling her to a share in the estate of Arturo as an heir

Held: No.

We note that in her comment to petitioner's motion private respondent raised, among others, the issue as to whether petitioner was still entitled to inherit from the decedent considering that she had secured a divorce in the U.S.A. and in fact had twice remarried. She also invoked the above quoted procedural rule. To this, petitioner replied that Arturo was a Filipino and as such remained legally married to her in spite of the divorce they obtained. Reading between the lines, the implication is that petitioner was no longer a Filipino citizen at the time of her divorce from Arturo. This should have prompted the trial court to conduct a hearing to establish her citizenship. The purpose of a hearing is to ascertain the truth of the matters in issue with the aid of documentary and testimonial evidence as well as the arguments of the parties either supporting or opposing the evidence. Instead, the lower court perfunctorily settled her claim in her favor by merely applying the ruling in Tenchavez v. Escaño.

Then in private respondent's motion to set aside and/or reconsider the lower court's decision she stressed that the citizenship of petitioner was relevant in the light of the ruling in Van Dorn v. Romillo Jr. that aliens may obtain divorces abroad, which may be recognized in the Philippines, provided they are valid according to their national law. She prayed therefore that the case be set for hearing. Petitioner opposed the motion but failed to squarely address the issue on her citizenship. The trial court did not grant private respondent's prayer for a hearing but proceeded to resolve her motion with the finding that both petitioner and Arturo were "Filipino citizens and were married in the Philippines." It maintained that their divorce obtained in 1954 in San Francisco, California, U.S.A., was not valid in Philippine jurisdiction. We deduce that the finding on their citizenship pertained solely to the time of their marriage as the trial court was not supplied with a basis to determine petitioner's citizenship at the time of their divorce. The doubt persisted as to whether she was still a Filipino citizen when their divorce was decreed. The trial court must have overlooked the materiality of this aspect. Once proved that she was no longer a Filipino citizen at the time of their divorce, Van Dorn would become applicable and petitioner could very well lose her right to inherit from Arturo.

In the present proceeding, petitioner's citizenship is brought anew to the fore by private respondent. She even furnishes the Court with the transcript of stenographic notes taken on 5 May 1995 during the hearing for the reconstitution of the original of a certain transfer certificate title as well as the issuance of new owner's duplicate copy thereof before another trial court. When asked whether she was an American citizen petitioner answered that she was since 1954.[ Significantly, the decree of divorce of petitioner and Arturo was obtained in the same year. Petitioner however did not bother to file a reply memorandum to erase the uncertainty about her citizenship at the time of their divorce, a factual issue requiring hearings to be conducted by the trial court. Consequently, respondent appellate court did not err in ordering the case returned to the trial court for further proceedings.

The decision of respondent Court of Appeals ordering the remand of the case to the court of origin for further proceedings and declaring null and void its decision holding petitioner Fe D. Quita and Ruperto T. Padlan as intestate heirs is AFFIRMED.

Republic v. Iyoy

September 21, 2005 G.R. No. 152577

Facts: The proceedings before the RTC commenced with the filing of a Complaint for declaration of nullity of marriage by respondent Crasus Iyoy on 25 March 1997. According to the said Complaint, respondent Crasus married Fely Ada Rosal-Iyoy on 16 December 1961 at Bradford Memorial Church, Jones Avenue, Cebu City. As a result of their union, they had five children – Crasus, Jr., Daphne, Debbie, Calvert, and Carlos – who are now all of legal ages. After the celebration of their marriage, respondent Crasus discovered that Fely was “hot-tempered, a nagger and extravagant.”

In 1984, Fely left the Philippines for the US, leaving all of their five children to the care of Crasus. Barely a year after Fely left for the US, respondent Crasus received a letter from her requesting that he sign the enclosed divorce papers; he disregarded the said request. Sometime in 1985, respondent Crasus learned, through the letters sent by Fely to their children, that Fely got married to an American, with whom she eventually had a child.

In 1987, Fely came back to Cebu with her American family but Crasus did not talk to Fely. Fely returned to the Philippines several times more: in 1990 (wedding of Crasus Jr.), 1992 (brain operation of 4th child, Calvert) and in 1995 for unknown reasons. Fely continued to live with her American family in

New Jersey and had been openly using the surname of her American husband “Micklus” in the Philippines and in the US. At the time the Complaint was filed, it had been 13 years since Fely left and abandoned respondent Crasus, and there was no more possibility of reconciliation between them.

Crasus also alleged in his Complaint that Fely’s acts brought clearly demonstrated her psychological incapacity to perform the essential obligations of marriage. Such incapacity, being incurable and continuing, constitutes a ground for declaration of nullity of marriage under Article 36, in relation to Articles 68, 70, and 72, of the Family Code of the Philippines.

Fely filed her Answer and Counter claim with the RTC asserting that she was already an American citizen since 1988 and was now married to Stephen Micklus. Fely refuted the other allegations made by respondent Crasus in his Complaint. She explained that she was no more hot-tempered than any

normal person, and she may had been indignant at respondent Crasus on certain occasions but it was because of the latter ’s drunkenness, womanizing, and lack of sincere effort to find employment and to contribute to the maintenance of their household. She could not have been extravagant since the family hardly had enough money for basic needs.

Indeed, Fely left for abroad for financial reasons as respondent Crasus had no job and what she was then earning as the sole breadwinner in the Philippines was insufficient to support their family. Although she left all of her children with respondent Crasus, she continued to provide financial support to them, as well as, to respondent Crasus. Subsequently, Fely was able to bring her children to the U.S.A., except for one, Calvert, who had to stay behind for medical reasons. While she did file for divorce from respondent Crasus, she denied having herself sent a letter to respondent Crasus requesting him to sign the enclosed divorce papers. After securing a divorce from respondent Crasus, Fely married her American husband and acquired American citizenship. She argued that her marriage to her American husband was legal because now being an American citizen, her status shall be governed by the law of her present nationality. She likewise prayed for the declaration of nullity of their marriage.

RTC promulgated its Judgment declaring the marriage of respondent Crasus and Fely null and void ab initio, and found that Fely exhibited unmistakable signs of such psychological incapacity to comply with her marital obligations.

Petitioner Republic, believing that the afore-quoted Judgment of the RTC was contrary to law and evidence, filed an appeal with the Court of Appeals. CA affirmed the decision of the RTC and stated that

Defendant secured a divorce from plaintiff-appellee abroad, has remarried, and is now permanently residing in the United States. Plaintiff-appellee categorically stated this as one of his reasons for seeking the declaration of nullity of their marriage

Article 26 of the Family Code provides:

“Art. 26.   All marriages solemnized outside the Philippines in accordance with the laws in force in the country where they were solemnized, and valid there as such, shall also be valid in this country, except those prohibited under Articles 35(1), (4), (5) and (6), 36, 37 and 38.

“WHERE A MARRIAGE BETWEEN A FILIPINO CITIZEN AND A FOREIGNER IS VALIDLY CELEBRATED AND A DIVORCE IS THEREAFTER VALIDLY OBTAINED ABROAD BY THE ALIEN SPOUSE CAPACITATING HIM OR HER TO REMARRY, THE FILIPINO SPOUSE SHALL LIKEWISE HAVE CAPACITY TO REMARRY UNDER PHILIPPINE LAW.”

Issue: Whether the CA is correct in applying Art. 26 of the Family Code

Held: No.

As it is worded, Article 26, paragraph 2, refers to a special situation wherein one of the couple getting married is a Filipino citizen and the other a foreigner at the time the marriage was celebrated. By its plain and literal interpretation, the said provision cannot be applied to the case of respondent Crasus and his wife Fely because at the time Fely obtained her divorce, she was still a Filipino citizen. Although the exact date was not established, Fely herself admitted in her Answer filed before the RTC that she obtained a divorce from respondent Crasus sometime after she left for the United States in 1984, after which she married her American husband in 1985. In the same Answer, she alleged that she had been an American citizen since 1988. At the time she filed for divorce, Fely was still a Filipino citizen, and pursuant to the nationality principle embodied in Article 15 of the Civil Code of the Philippines, she was still bound by Philippine laws on family rights and duties, status, condition, and legal capacity, even when she was already living abroad. Philippine laws, then and even until now, do not allow and recognize divorce between Filipino spouses. Thus, Fely could not have validly obtained a divorce from respondent Crasus.

Republic vs Iyoy (G.R. No. 152577)

Facts: The case is a petition for review by the RP represented by the Office of the Solicitor General on certiorari praying for thereversal of the decision of the CA dated July 30, 2001 affirming the judgment of the RTC declaring the marriage of Crasus L. Iyoy(respondent) and Ada Rosal-Iyoy null and void ba -sed on Article 36.

On December 16, 1961 Crasus Iyoy and Ada Rosal-Iyoy married each other, they had 5 children. In 1984, Fely went to the US, inthe same year she sent letters to Crasus asking him to sign divorce papers. In 1985, Crasus learned that Fely married an Americanand had a child. Fely went back to the Philip-pines on several occasions, during one she attended the marriage of one of her children inwhich she used her husband’s last name as hers in the invitation.

March 25, 1997, Crasus filed a complaint for declaration of nullity alleging that Fely’s acts brought “danger and dishonor” to the family and were manifesta-tions of her psychological incapacity. Crasus submitted his testimony, the certification of the recording of their marriage contract, and the invitation where Fely used her newhusband’s last name as evidences.

Fely denied the claims and asserted that Crasus was a drunkard, womanizer, had no job, and thatsince 1988 she was already an American citizen and not covered by our laws. The RTC found the evidences sufficient and granted thedecree; it was affirmed in the CA.

Issue: Does abandonment and sexual infidelity per se constitute psychological incapacity?

Held: The evidences presented by the respondent fail to establish psychological incapacity.

Furthermore, Article 36 “contemplates downright incapacity or inability to take cognizance of and to assume the basic marital obligations; not a mere refu-sal, neglect or difficulty, much less, ill will, on the part of the errant spouse. Irreconcilable differences, conflicting personalities, emotional immaturity and ir -responsibility, physical abuse, habitual alcoholism, sexual infidelity or perversion, and abandonment, by themselves, also do not warrant a finding of psy-chological incapacity under the said Article.”

Finally, Article 36 “is not to be confused with a divorce law thatcuts the marital bond at the time the causes therefore manifest themselves. It refers to a se-rious psychological illness afflicting aparty even before the celebration of marriage. It is a malady so grave and so permanent as to deprive one of aware -ness of the duties and responsibilities of the matrimonial bond one is about to assume.”

RULING: What constitute a psychological incapacity?

Citing Santos v CA, the Court stated that “psychological incapacity refers to no less than a mental incapacity that causes a party to be truly cognitive of the

basic marital covenants that concomitantly must be assumed and discharged by the parties to the marriage”.

The only evidence presented by Crasus was his testimony which is put in question for being self-serving. The evidence is not enough to convince the Court that Fely had such grave mental illness that prevented her from assuming the essential obligations of marriage.

It is worthy to emphasize that Art 36 contemplates downright incapacity or inability to take cognizance of and to assume basic marital obligations; not a mere refusal, neglect or difficulty, much less, ill will, on the part of the errant spouse.

The evidence may have proven that Fely committed acts that hurt and embarrassed respondent and the rest of the family. Her hot-temper, nagging, and extravagance; her abandonment of respondent; her marriage to an Am citizen; and even her flaunting of her American family may be indeed manifestations of her alleged incapacity, nonetheless, the root cause for such was not identified. If the root cause f the incapacity was not identified, then, it cannot be sa-tisfactorily established as a psychological

Llorente v. Ca

G.R. No. 124371 November 23, 2000

Facts: Lorenzo N. Llorente was an enlisted serviceman of the United States Navy from March 10, 1927 to September 30, 1957.

On February 22, 1937, Lorenzo and petitioner Paula Llorentewere married before a parish priest, Roman Catholic Church, in Nabua, Camarines Sur. Before the outbreak of the Pacific War, Lorenzo departed for the United States and Paula stayed in the conjugal home in barrio Antipolo, Nabua, Camarines Sur. On November 30, 1943, Lorenzo was admitted to United States citizenship and Certificate of Naturalization No. 5579816 was issued in his favor by the United States District Court, Southern District of New York.

Upon the liberation of the Philippines by the American Forces in 1945, Lorenzo was granted an accrued leave by the U. S. Navy, to visit his wife and he

visited the Philippines. He discovered that his wife Paula was pregnant and was “living in” and having an adulterous relationship with his brother, Ceferino Llorente.

On December 4, 1945, Paula gave birth to a boy registered in the Office of the Registrar of Nabua as “Crisologo Llorente,” with the certificate stating that

the child was not legitimate and the line for the father’s name was left blank.

Lorenzo refused to forgive Paula and live with her. In fact, on February 2, 1946, the couple drew a written agreement to the effect that (1) all the family

allowances allotted by the United States Navy as part of Lorenzo’s salary and all other obligations for Paula’s daily maintenance and support would be suspended; (2) they would dissolve their marital union in accordance with judicial proceedings; (3) they would make a separate agreement regarding their conjugal property acquired during their marital life; and (4) Lorenzo would not prosecute Paula for her adulterous act since she voluntarily admitted her fault and agreed to separate from Lorenzo peacefully. The agreement was signed by both Lorenzo and Paula and was witnessed by Paula ’s father and stepmother. The agreement was notarized by Notary Public Pedro Osabel.

Lorenzo returned to the United States and on November 16, 1951 filed for divorce with the Superior Court of the State of California in and for the County of San Diego. Paula was represented by counsel, John Riley, and actively participated in the proceedings. On November 27, 1951, the Superior Court of the

State of California, for the County of San Diego found all factual allegations to be true and issued an interlocutory judgment of divorce. On December 4, 1952, the divorce decree became final.

Lorenzo returned to Manila and on January 16, 1958, Lorenzo married Alicia F. Llorente in Manila wherein from 1958 to 1985, Lorenzo and Alicia lived together as husband and wife. Their twenty-five (25) year union produced three children, Raul, Luz and Beverly, all surnamed Llorente.

On March 13, 1981, Lorenzo executed a Last Will and Testament wherein he bequeathed all his property to Alicia and their three children.

On December 14, 1983, Lorenzo filed with the RTC of Iriga, Camarines Sur, a petition for the probate and allowance of his last will and testament wherein Lorenzo moved that Alicia be appointed Special Administratrix of his estate. At first the RTC denied probate of the will for the reason that Lorenzo was still alive but it eventually admitted the will for probate. However, before the proceedings could be terminated, Lorenzo died.

On September 4, 1985, Paula filed with the same court a petition for letters of administration over Lorenzo ’s estate in her favor. Paula contended (1) that

she was Lorenzo’s surviving spouse, (2) that the various property were acquired during their marriage, (3) that Lorenzo ’s will disposed of all his property in favor of Alicia and her children, encroaching on her legitime and 1/2 share in the conjugal property.

The Court gave due course to Paula’s petition. The trial court issued a decision finding the divorce decree granted to the late Lorenzo Llorente is void and inapplicable in the Philippines, therefore the marriage he contracted with Alicia Fortunato on January 16, 1958 at Manila is likewise void. This being so the petition of Alicia F. Llorente for the issuance of letters testamentary was denied. The RTC found the instrinsic disposition of the will void and declared Paula

as the legal administrator of the estate, declared her entitled as conjugal partner, ½ of their conjugal properties, and as a primary compulsory heir, Paula is also entitled to 1/3 of the estate, 1/3 for the illegitimate children, Raul, Luz an Beverly for them to partition in equal shares and also entitled to the remaining free portion in equal shares.

Issue: Whether the divorce decree granted to Lorenzo by the California court is valid under the Philippine courts

Held: Yes.

The fact that the late Lorenzo N. Llorente became an American citizen long before and at the time of: (1) his divorce from Paula; (2) marriage to Alicia; (3) execution of his will; and (4) death, is duly established, admitted and undisputed.

Thus, as a rule, issues arising from these incidents are necessarily governed by foreign law.

The Civil Code clearly provides:

“Art. 15.   Laws relating to family rights and duties, or to the status, condition and legal capacity of persons are  binding upon citizens of the Philippines, even though living abroad.

“Art. 16.   Real property as well as personal property is subject to the law of the country where it is situated.

“However, intestate and testamentary succession, both with respect to the order of succession and to the amount of successional rights and to the intrinsic validity of testamentary provisions, shall be regulated by the national law of the person whose succession is under consideration, whatever may be the

nature of the property and regardless of the country wherein said property may be found.” 

True, foreign laws do not prove themselves in our jurisdiction and our courts are not authorized to take judicial notice of them. Like any other fact, they must be alleged and proved.

While the substance of the foreign law was pleaded, the Court of Appeals did not admit the foreign law. The Court of Appeals and the trial court called to the fore the renvoi doctrine, where the case was “referred back” to the law of the decedent’s domicile, in this case, Philippine law.

We note that while the trial court stated that the law of New York was not sufficiently proven, in the same breath it made the categorical, albeit equally unproven statement that “American law follows the ‘domiciliary theory’ hence, Philippine law applies when determining the validity of Lorenzo’s will.

First, there is no such thing as one American law. The "national law" indicated in Article 16 of the Civil Code cannot possibly apply to general American law. There is no such law governing the validity of testamentary provisions in the United States. Each State of the union has its own law applicable to its citizens and in force only within the State. It can therefore refer to no other than the law of the State of which the decedent was a resident. Second, there is no showing that the application of the renvoi doctrine is called for or required by New York State law.

The trial court held that the will was intrinsically invalid since it contained dispositions in favor of Alice, who in the trial court’s opinion was a mere paramour. The trial court threw the will out, leaving Alice, and her two children, Raul and Luz, with nothing.

The Court of Appeals also disregarded the will. It declared Alice entitled to one half (1/2) of whatever property she and Lorenzo acquired during their cohabitation, applying Article 144 of the Civil Code of the Philippines.

The hasty application of Philippine law and the complete disregard of the will, already probated as duly executed in accordance with the formalities of Philippine law, is fatal, especially in light of the factual and legal circumstances here obtaining.

With respect ro the validity of the foreign divorce, In Van Dorn v. Romillo, Jr. we held that owing to the nationality principle embodied in Article 15 of the Civil Code, only Philippine nationals are covered by the policy against absolute divorces, the same being considered contrary to our concept of public policy and morality. In the same case, the Court ruled that aliens may obtain divorces abroad, provided they are valid according to their national law.

Citing this landmark case, the Court held in Quita v. Court of Appeals, that once proven that respondent was no longer a Filipino citizen when he obtained the divorce from petitioner, the ruling in Van Dorn would become applicable and petitioner could “very well lose her right to inherit” from him.

In Pilapil v. Ibay-Somera, we recognized the divorce obtained by the respondent in his country, the Federal Republic of Germany. There, we stated that divorce and its legal effects may be recognized in the Philippines insofar as respondent is concerned in view of the nationality principle in our civil law on the status of persons.

For failing to apply these doctrines, the decision of the Court of Appeals must be reversed. We hold that the divorce obtained by Lorenzo H. Llorente from his first wife Paula was valid and recognized in this jurisdiction as a matter of comity. Now, the effects of this divorce (as to the succession to the estate of the decedent) are matters best left to the determination of the trial court.

PAULA T. LLORENTE, petitioner, VS. COURT OF APPEALS and ALICIA F. LLORENTE

FACTS: Lorenzo Llorente and petitioner Paula Llorente were married in 1937 in the Philippines. Lorenzo was an enlisted serviceman of the US Navy. Soon after, he left for the US where through naturalization, he became a US Citizen. Upon his visitation of his wife, he discovered that she was living with his bro-ther and a child was born. The child was registered as legitimate but the name of the father was left blank. Llorente filed a divorce in California, which later on became final. He married Alicia and they lived together for 25 years bringing 3 children. He made his last will and testament stating that all his properties will be given to his second marriage. He filed a petition of probate that made or appointed Alicia his special administrator of his estate.

Before the proceeding could be terminated, Lorenzo died. Paula filed a letter of administration over Llorente’s estate. The trial granted the letter and denied the motion for reconsideration. An appeal was made to the Court of Appeals, which affirmed and modified the judgment of the Trial Court that she be decla-red co-owner of whatever properties, she and the deceased, may have acquired during their 25 years of cohabitation.

ISSUE: Whether or not the National Law shall apply.

RULING: Lorenzo Llorente was already an American citizen when he divorced Paula. Such was also the situation when he married Alicia and executed his will. As stated in Article 15 of the civil code, aliens may obtain divorces abroad, provided that they are validly required in their National Law. Thus the di-vorce obtained by Llorente is valid because the law that governs him is not Philippine Law but his National Law since the divorce was contracted after he became an American citizen. Furthermore, his National Law allowed divorce.

The case was remanded to the court of origin for determination of the intrinsic validity of Lorenzo Llorente’s will and determination of the parties’ successio-nal rights allowing proof of foreign law.

Republic vs. Orbecido GR NO. 154380, October 5, 2005

FACTS: Cipriano Orbecido III was married with Lady Myros Villanueva on May 24, 1981 at the United Church of Christ in the Philippines in Ozamis City. They had a son and a daughter named Kristoffer and Kimberly, respectively. In 1986, the wife left for US bringing along their son Kristoffer. A few years la -ter, Orbecido discovered that his wife had been naturalized as an American citizen and learned from his son that his wife sometime in 2000 had obtained a divorce decree and married a certain Stanley. He thereafter filed with the trial court a petition for authority to remarry invoking Paragraph 2 of Article 26 of the Family Code.

ISSUE: Whether or not Orbecido can remarry under Article 26 of the Family Code.

HELD: The court ruled that taking into consideration the legislative intent and applying the rule of reason, Article 26 Par.2 should be interpreted to include cases involving parties who, at the time of the celebration of the marriage were Filipino citizens, but later on, one of them becomes naturalized as a foreign citizen and obtains a divorce decree. The Filipino spouse should likewise be allowed to remarry as if the other party were a foreigner at the time of the so -lemnization of the marriage.

Hence, the court’s unanimous decision in holding Article 26 Par 2 be interpreted as allowing a Filipino citizen who has been divorced by a spouse who had acquired a citizenship and remarried, also to remarry under Philippine law.

Grace J. Garcia-Recio vs Rederick A. Recio GR NO. 138322, Oct. 2, 2002 | 366 SCRA 437

FACTS: Rederick A. Recio, a Filipino, was married to Editha Samson, an Australian Citizen, in Malabon, Rizal on March 1, 1987. They lived as husband and wife in Australia. However, an Australian family court issued purportedly a decree of divorce, dissolving the marriage of Rederick and Editha on May 18, 1989.

On January 12, 1994, Rederick married Grace J. Garcia where it was solemnized at Our lady of Perpetual Help Church, Cabanatuan City. Since October 22, 1995, the couple lived separately without prior judicial dissolution of their marriage. As a matter of fact, while they were still in Australia, their conjugal assets were divided on May 16, 1996, in accordance with their Statutory Declarations secured in Australia.

Grace filed a Complaint for Declaration of Nullity of Marriage on the ground of bigamy on March 3, 1998, claiming that she learned only in November 1997, Rederick’s marriage with Editha Samson.

ISSUE: Whether the decree of divorce submitted by Rederick Recio is admissible as evidence to prove his legal capacity to marry petitioner and absolved him of bigamy.

HELD: The nullity of Rederick’s marriage with Editha as shown by the divorce decree issued was valid and recognized in the Philippines since the re-

spondent is a naturalized Australian. However, there is absolutely no evidence that proves respondent ’s legal capacity to marry petitioner though the for-mer presented a divorce decree. The said decree, being a foreign document was inadmissible to court as evidence primarily because it was not authenti-cated by the consul/ embassy of the country where it will be used.

Under Sections 24 and 25 of Rule 132, a writing or document may be proven as a public or official record of a foreign country by either:

(1) an official publication or

(2) a copy thereof attested by the officer having legal custody of the document. If the record is not kept in the Philippines, such copy must be:

(a) accompanied by a certificate issued by the proper diplomatic or consular officer in the Philippine foreign service stationed in the foreign country in which the record is kept and

(b) authenticated by the seal of his office.

Thus, the Supreme Court remands the case to the Regional Trial Court of Cabanatuan City to receive or trial evidence that will conclusively prove respon -dent’s legal capacity to marry petitioner and thus free him on the ground of bigamy.

Republic v. Obrecido

October 5, 2005 G.R. No. 154380

Facts: On May 24, 1981, Cipriano Orbecido III married Lady Myros M. Villanueva at the United Church of Christ in the Philippines in Lam-an, Ozamis City. Their marriage was blessed with a son and a daughter, Kristoffer Simbortriz V. Orbecido and Lady Kimberly V. Orbecido.

In 1986, Cipriano’s wife left for the United States along their son Kristoffer. A few years later, Cipriano discovered that his wife had been naturalized as an American citizen.

Sometime in 2000, Cipriano learned from his son that his wife had obtained a divorce decree and then married a certain Innocent Stanley. She, Stanley and her child by him currently in San Gabriel, California.

Cipriano thereafter filed with the trial court a petition for authority to remarry invoking Paragraph 2 of Article 26 of the Family Code. No opposition was filed. Finding merit in the petition, the court granted the same. The Republic, herein petitioner, through the Office of the Solicitor General (OSG), sought reconsideration but it was denied.

Issue: Whether Cipriano can remarry under Art. 26 of the Family Code

Held: Yes.

Paragraph 2 of Article 26 traces its origin to the 1985 case of Van Dorn v. Romillo, Jr The Van Dorn case involved a marriage between a Filipino citizen and a foreigner. The Court held therein that a divorce decree validly obtained by the alien spouse is valid in the Philippines, and consequently, the Filipino spouse is capacitated to remarry under Philippine law.

Does the same principle apply to a case where at the time of the celebration of the marriage, the parties were Filipino citizens, but later on, one of them obtains a foreign citizenship by naturalization?

The jurisprudential answer lies latent in the 1998 case of Quita v. Court of Appeals. In Quita, the parties were, as in this case, Filipino citizens when they got married. The wife became a naturalized American citizen in 1954 and obtained a divorce in the same year. The Court therein hinted, by way of obiter dictum, that a Filipino divorced by his naturalized foreign spouse is no longer married under Philippine law and can thus remarry.

Thus, taking into consideration the legislative intent and applying the rule of reason, we hold that Paragraph 2 of Article 26 should be interpreted to include cases involving parties who, at the time of the celebration of the marriage were Filipino citizens, but later on, one of them becomes naturalized as a foreign citizen and obtains a divorce decree. The Filipino spouse should likewise be allowed to remarry as if the other party were a foreigner at the time of the solemnization of the marriage. To rule otherwise would be to sanction absurdity and injustice. Where the interpretation of a statute according to its exact and literal import would lead to mischievous results or contravene the clear purpose of the legislature, it should be construed according to its spirit and reason, disregarding as far as necessary the letter of the law. A statute may therefore be extended to cases not within the literal meaning of its terms, so long as they come within its spirit or intent.

If we are to give meaning to the legislative intent to avoid the absurd situation where the Filipino spouse remains married to the alien spouse who, after obtaining a divorce is no longer married to the Filipino spouse, then the instant case must be deemed as coming within the contemplation of Paragraph 2 of Article 26.

In view of the foregoing, we state the twin elements for the application of Paragraph 2 of Article 26 as follows:

1.                                   There is a valid marriage that has been celebrated between a Filipino citizen and a foreigner;

2.                                   A valid divorce is obtained abroad by the alien spouse capacitating him or her to remarry.

The reckoning point is not the citizenship of the parties at the time of the celebration of the marriage, but their citizenship at the time a valid divorce is obtained abroad by the alien spouse capacitating the latter to remarry.

In this case, when Cipriano’s wife was naturalized as an American citizen, there was still a valid marriage that has been celebrated between her and Cipriano. As fate would have it, the naturalized alien wife subsequently obtained a valid divorce capacitating her to remarry. Clearly, the twin requisites for the application of Paragraph 2 of Article 26 are both present in this case. Thus Cipriano, the “divorced” Filipino spouse, should be allowed to remarry.

Grace J. Garcia-Recio vs Rederick A. Recio GR NO. 138322, Oct. 2, 2002 | 366 SCRA 437

FACTS: Rederick A. Recio, a Filipino, was married to Editha Samson, an Australian Citizen, in Malabon, Rizal on March 1, 1987. They lived as husband and wife in Australia. However, an Australian family court issued purportedly a decree of divorce, dissolving the marriage of Rederick and Editha on May 18, 1989.

On January 12, 1994, Rederick married Grace J. Garcia where it was solemnized at Our lady of Perpetual Help Church, Cabanatuan City. Since October 22, 1995, the couple lived separately without prior judicial dissolution of their marriage. As a matter of fact, while they were still in Australia, their conjugal assets were divided on May 16, 1996, in accordance with their Statutory Declarations secured in Australia.

Grace filed a Complaint for Declaration of Nullity of Marriage on the ground of bigamy on March 3, 1998, claiming that she learned only in November 1997, Rederick’s marriage with Editha Samson.

ISSUE: Whether the decree of divorce submitted by Rederick Recio is admissible as evidence to prove his legal capacity to marry petitioner and absolved him of bigamy.

HELD: The nullity of Rederick’s marriage with Editha as shown by the divorce decree issued was valid and recognized in the Philippines since the re-

spondent is a naturalized Australian. However, there is absolutely no evidence that proves respondent ’s legal capacity to marry petitioner though the for-mer presented a divorce decree. The said decree, being a foreign document was inadmissible to court as evidence primarily because it was not authenti-cated by the consul/ embassy of the country where it will be used.

Under Sections 24 and 25 of Rule 132, a writing or document may be proven as a public or official record of a foreign country by either:

(1) an official publication or

(2) a copy thereof attested by the officer having legal custody of the document. If the record is not kept in the Philippines, such copy must be:

(a) accompanied by a certificate issued by the proper diplomatic or consular officer in the Philippine foreign service stationed in the foreign country in which the record is kept and

(b) authenticated by the seal of his office.

Thus, the Supreme Court remands the case to the Regional Trial Court of Cabanatuan City to receive or trial evidence that will conclusively prove respon -dent’s legal capacity to marry petitioner and thus free him on the ground of bigamy.

Garcia v. Recio

G.R. No. 138322   October 2, 2001

Facts: Rederick A. Recio, Filipino, was married to Editha Samson, an Australian citizen, in Malabon, Rizal, on March 1, 1987. They lived together as husband and wife in Australia. On May 18, 1989, a decree of divorce, purportedly dissolving the marriage, was issued by an Australian family court.

On June 26, 1992, respondent became an Australian citizen, as shown by a “Certificate of Australian Citizenship” issued by the Australian government.

Petitioner, Grace Garcia -- a Filipina -- and respondent were married on January 12, 1994 in Our Lady of Perpetual Help Church in Cabanatuan City. In their application for a marriage license, respondent was declared as “single” and “Filipino.”

Starting October 22, 1995, petitioner and respondent lived separately without prior judicial dissolution of their marriage. While the two were still in Australia, their conjugal assets were divided on May 16, 1996, in accordance with their Statutory Declarations secured in Australia.

On March 3, 1998, petitioner filed a Complaint for Declaration of Nullity of Marriage in the court a quo, on the ground of bigamy -- respondent allegedly had a prior subsisting marriage at the time he married her on January 12, 1994. She claimed that she learned of respondent’s marriage to Editha Samson only in November, 1997.

In his Answer, He contended that his first marriage to an Australian citizen had been validly dissolved by a divorce decree obtained in Australia in 1989 thus, he was legally capacitated to marry petitioner in 1994.

On July 7, 1998 -- or about 5 years after the couple’s wedding and while the suit for the declaration of nullity was pending -- respondent was able to secure

a divorce decree from a family court in Sydney, Australia because the “marriage ha[d] irretrievably broken down.”

Respondent prayed in his Answer that the Complaint be dismissed on the ground that it stated no cause of action.

Issue: Whether the divorce decree obtained in Autralia should be automatically recognized before the Philippine courts

Held: No

Petitioner assails the trial court's recognition of the divorce between respondent and Editha Samson. Citing Adong v. Cheong Seng Gee, petitioner argues that the divorce decree, like any other foreign judgment, may be given recognition in this jurisdiction only upon proof of the existence of (1) the foreign law allowing absolute divorce and (2) the alleged divorce decree itself. She adds that respondent miserably failed to establish these elements.

Petitioner adds that, based on the first paragraph of Article 26 of the Family Code, marriages solemnized abroad are governed by the law of the place where they were celebrated (the lex loci celebrationist). In effect, the Code requires the presentation of the foreign law to show the conformity of the marriage in question to the legal requirements of the place where the marriage was performed.

At the outset, we lay the following basic legal principles as the take-off points for our discussion. Philippine law does not provide for absolute divorce; hence, our courts cannot grant it. A marriage between two Filipinos cannot be dissolved even by a divorce obtained abroad, because of Articles 1522 and 1723 of the Civil Code.24 In mixed marriages involving a Filipino and a foreigner, Article 2625 of the Family Code allows the former to contract a subsequent marriage in case the divorce is "validly obtained abroad by the alien spouse capacitating him or her to remarry." A divorce obtained abroad by a couple, who are both aliens, may be recognized in the Philippines, provided it is consistent with their respective national laws.

A comparison between marriage and divorce, as far as pleading and proof are concerned, can be made. Van Dorn v. Romillo Jr. decrees that "aliens may obtain divorces abroad, which may be recognized in the Philippines, provided they are valid according to their national law."Therefore, before a foreign divorce decree can be recognized by our courts, the party pleading it must prove the divorce as a fact and demonstrate its conformity to the foreign law allowing it. Presentation solely of the divorce decree is insufficient.

Burden of Proving Australian Law

Petitioner insists that before a divorce decree can be admitted in evidence, it must first comply with the registration requirements under Articles 11, 13 and 52 of the Family Code. These articles read as follows:

"ART. 11. Where a marriage license is required, each of the contracting parties shall file separately a sworn application for such license with the proper local civil registrar which shall specify the following:

x x x         x x x         x x x

"(5) If previously married, how, when and where the previous marriage was dissolved or annulled;

x x x           x x x           x x x

"ART. 13. In case either of the contracting parties has been previously married, the applicant shall be required to furnish, instead of the birth of baptismal certificate required in the last preceding article, the death certificate of the deceased spouse or the judicial decree of annulment or declaration of nullity of his or her previous marriage. x x x.

"ART. 52. The judgment of annulment or of absolute nullity of the marriage, the partition and distribution of the properties of the spouses, and the delivery of the children's presumptive legitimes shall be recorded in the appropriate civil registry and registries of property; otherwise, the same shall not affect their persons."

Respondent, on the other hand, argues that the Australian divorce decree is a public document – a written official act of an Australian family court. Therefore, it requires no further proof of its authenticity and due execution.

Before a foreign judgment is given presumptive evidentiary value, the document must first be presented and admitted in evidence. A divorce obtained abroad is proven by the divorce decree itself. Indeed the best evidence of a judgment is the judgment itself. The decree purports to be a written act or record of an act of an officially body or tribunal of a foreign country.

Under Sections 24 and 25 of Rule 132, on the other hand, a writing or document may be proven as a public or official record of a foreign country by either (1) an official publication or (2) a copy thereof attested33 by the officer having legal custody of the document. If the record is not kept in the Philippines, such copy must be (a) accompanied by a certificate issued by the proper diplomatic or consular officer in the Philippine foreign service stationed in the foreign country in which the record is kept and (b) authenticated by the seal of his office.

The divorce decree between respondent and Editha Samson appears to be an authentic one issued by an Australian family court. However, appearance is not sufficient; compliance with the aforemetioned rules on evidence must be demonstrated.

Fortunately for respondent's cause, when the divorce decree of May 18, 1989 was submitted in evidence, counsel for petitioner objected, not to its admissibility, but only to the fact that it had not been registered in the Local Civil Registry of Cabanatuan City. The trial court ruled that it was admissible, subject to petitioner's qualification. Hence, it was admitted in evidence and accorded weight by the judge. Indeed, petitioner's failure to object properly rendered the divorce decree admissible as a written act of the Family Court of Sydney, Australia.

Compliance with the quoted articles (11, 13 and 52) of the Family Code is not necessary; respondent was no longer bound by Philippine personal laws after he acquired Australian citizenship in 1992. Naturalization is the legal act of adopting an alien and clothing him with the political and civil rights belonging to a citizen. Naturalized citizens, freed from the protective cloak of their former states, don the attires of their adoptive countries. By becoming an Australian, respondent severed his allegiance to the Philippines and the vinculum juris that had tied him to Philippine personal laws.

Burden of Proving Australian Law

Respondent contends that the burden to prove Australian divorce law falls upon petitioner, because she is the party challenging the validity of a foreign judgment. He contends that petitioner was satisfied with the original of the divorce decree and was cognizant of the marital laws of Australia, because she had lived and worked in that country for quite a long time. Besides, the Australian divorce law is allegedly known by Philippine courts: thus, judges may take judicial notice of foreign laws in the exercise of sound discretion.

We are not persuaded. The burden of proof lies with "the party who alleges the existence of a fact or thing necessary in the prosecution or defense of an action." In civil cases, plaintiffs have the burden of proving the material allegations of the complaint when those are denied by the answer; and defendants have the burden of proving the material allegations in their answer when they introduce new matters. Since the divorce was a defense raised by respondent, the burden of proving the pertinent Australian law validating it falls squarely upon him.

It is well-settled in our jurisdiction that our courts cannot take judicial notice of foreign laws.Like any other facts, they must be alleged and proved. Australian marital laws are not among those matters that judges are supposed to know by reason of their judicial function. The power of judicial notice must be exercised with caution, and every reasonable doubt upon the subject should be resolved in the negative.

Corpuz vs. Sto. Tomas and Sol Gen G.R. No. 186571, 11 August 2010

Facts: Petitioner was a former Filipino citizen who acquired Canadian citizenship through naturalization. He was married to the respondent but was shocked of the infidelity on the part of his wife. He went back to Canada and filed a petition for divorce and was granted. Desirous to marry another woman he now loved, he registered the divorce decree in the Civil Registry Office and was informed that the foreign decree must first be judicially recognized by a com -petent Philippine court. Petitioner filed for judicial recognition of foreign divorce and declaration of marriage as dissolved with the RTC where respondent failed to submit any response. The RTC denied the petition on the basis that the petitioner lacked locus standi. Thus, this case was filed before the Court.

Issues: WON the second paragraph of Art 26 of the FC extends to aliens the right to petition a court of this jurisdiction fro the recognition of a foreign di -vorce decree.

Decision: The alien spouse cannot claim under the second paragraph of Art 26 of the Family Code because the substantive right it establishes is in fa -vour of the Filipino spouse. Only the Filipino spouse can invoke the second par of Art 26 of the Family Code.

The unavailability of the second paragraph of Art 26 of the Family Code to aliens does not necessarily strip the petitioner of legal interest to petition the RTC for the recognition of his foreign divorce decree. The petitioner, being a naturalized Canadian citizen now, is clothed by the presumptive evidence of the authenticity of foreign divorce decree with conformity to alien’s national law.

The Pasig City Civil Registry acted out of line when it registered the foreign decree of divorce on the petitioner and respondent ’s marriage certificate without judicial order recognizing the said decree. The registration of the foreign divorce decree without the requisite judicial recognition is void.

The petition for review on certiorari is granted, the RTC decision is reversed and Court ordered t6he remand of the case to the trial court for further procee -dings in light of the ruling.

Republic of the Philippines vs. CA

(1993)

FACTS: Hughes is married to Lenita, a Filipina who was later naturalized as an American citizen. They filed a petition to adopt the 3 nephews and niece of Lenita, which was granted.

HELD: Hughes is not qualified to adopt since he does not fall under the exceptions in Art. 184 of the Family Code. While Lenita, as a former Filipino, is qualified to adopt under that provision, the adoption decree still cannot be granted because of the requirement in Art. 185 that spouses must jointly adopt. They cannot do this in CAB because Hughes is not qualified under the law.

REPUBLIC OF THE PHILIPPINES vs CA G.R. No. 95551 March 20, 1997

On June 25, 1990, the spouses Samuel R. Dye, Jr. and Rosalina Due Dye filed a petition before the Regional Trial Court of Angeles City 1 to adopt Maricel R. Due and Alvin R. Due, ages 13 and 12 years old, respectively, younger siblings of Rosalina. Samuel R. Dye, Jr. a member of the United States Air Force, is an American citizen who resided at the Clark Air Base in Pampanga. His wife Rosalina is a former Filipino who became a naturalized American. They have two children. Both Maricel and Alvin Due, as well as their natural parents, gave their consent to the adoption.

After trial, the lower court rendered its decision on September 10, 1990 granting the petition and declaring Alvin and Maricel to be the children of the spouses Dye by adoption. 2 Respondent Regional Trial Court disregarded the sixteen-year age gap requirement of the law, the spouses being only fifteen years and three months and fifteen years and nine months older than Maricel Due, on the ground that a literal implementation of the law would defeat the very philosophy behind adoption statutes, namely, to promote the welfare of a child. 3 The court also found that the petitioning spouses are mentally and physically fit to adopt, possess good moral character, sufficient financial capability and love and affection for the intended adoptees.

The Republic filed this petition for review on a pure question of law, contending that the spouses Dye are not qualified under the law to adopt Maricel and Alvin Due.

The Court finds the petition meritorious and hereby grants it.

As a general rule, aliens cannot adopt Filipino citizens as this is proscribed under Article 184 of the Family Code which states:

Art. 184. The following persons may not adopt:

(3) An alien, except:

(a) A former Filipino citizen who seeks to adopt a relative by consanguinity;

(b) One who seeks to adopt the legitimate child of his or her Filipino spouse; or

(c) One who is married to a Filipino citizen and seeks to adopt jointly with his or her spouse a relative by consanguinity of the latter.

Aliens not included in the foregoing exceptions may adopt Filipino children in accordance with the rules on inter-country adoption as may be provided by law.

Samuel Robert Dye, Jr. who is an American and, therefore, an alien is disqualified from adopting the minors Maricel and Alvin Due because he does not fall under any of the three aforequoted exceptions laid down by the law. He is not a former Filipino citizen who seeks to adopt a relative by consanguinity. Nor does he seek to adopt his wife's legitimate child. Although he seeks to adopt with his wife her relatives by consanguinity, he is not married to a Filipino citi-zen, for Rosalina was already a naturalized American at the time the petition was filed, thus excluding him from the coverage of the exception. The law here

does not provide for an alien who is married to a former Filipino citizen seeking to adopt jointly with his or her spouse a relative by consanguinity, as an ex -ception to the general rule that aliens may not adopt.

On her own, Rosalina Dye cannot adopt her brother and sister for the law mandates joint adoption by husband and wife, subject to exceptions. Article 29 of Presidential Decree No. 603 (Child and Youth Welfare Code) retained the Civil Code provision 4 that husband and wife may jointly adopt. The Family Code amended this rule by scrapping the optional character of joint adoption and making it now mandatory. Article 185 of the Family Code provides:

Art. 185. Husband and wife must adopt, except in the following cases:

(1) When one spouse seeks to adopt his own illegitimate child;

(2) When one spouse seeks to adopt the legitimate child of the other.

None of the above exceptions applies to Samuel and Rosalina Dye, for they did not petition to adopt the latter's child but her brother and sister.

The Court has previously recognized the ineligibility of a similarly situated alien husband with a former Filipino wife seeking to adopt the latter's nephews and niece in the case of Republic v. Court of Appeals. 5 Although the wife in said case was qualified to adopt under Article 184, paragraph 3 (a), she being a former Filipino who seeks to adopt a relative by consanguinity, she could not jointly adopt with her husband under Article 185 because he was an alien in-eligible to adopt here in the Philippines.

We are not unmindful of the main purpose of adoption statutes, which is the promotion of the welfare of children. Accordingly, the law should be construed liberally, in a manner that will sustain rather than defeat said purpose. 6 The law must also be applied with compassion, understanding and less severity in view of the fact that it is intended to provide homes, love, care and education for less fortunate children. 7 Regrettably, the Court is not in a position to affirm the trial court's decision favoring adoption in the case at bar, for the law is clear and it cannot be modified without violating the proscription against judicial legislation. Until such time however, that the law on the matter is amended, we cannot sustain the respondent-spouses' petition for adoption.

WHEREFORE, the instant petition is hereby GRANTED. The Decision of the Regional Trial Court of Angeles City in Special Proceeding No. 4203 (In the Matter of the Petition for Adoption of the minors Maricel R Due and Alvin R. Due), dated September 10, 1990 is REVERSED AND SET ASIDE. SO ORDE-RED.

Uggi Lindamand Therkelsen vs. Republic

(1964)

FACTS: Therkelsen (a German) and his wife Erlinda (a Filipino), filed a petition to adopt Erlinda ’s natural child. The application was denied on the ground that an alien cannot adopt a Filipino unless the adoption would make the Filipino minor a citizen of the alien’s country.

HELD: The application should be granted. Being a permanent resident here, Therkelsen is not disqualified to adopt under our laws; to deny the application on the above stated ground would be to impose a further requisite on adoptions by aliens beyond those required by law. The citizenship of the adopter is a matter political, not civil in nature, and the ways in which it should be conferred lay outside the ambit of the Civil Code.

Therkelsen vs Republic G.R. No. L-21951 November 27, 1964

FACTS: The petitioners are husband and wife who were married on June 2, 1962, or barely a year ago. The minor sought to be adopted, born on February 16, 1960, is the natural child of petitioner wife. His father was Charles Joseph Week, who abandoned mother and child after the latter's birth. He is said to have gone back to the United States. Except for the legal impediment hereinafter to be mentioned, the facts before the Court may warrant the approval of the adoption sought herein. Petitioner husband is a Danish subject, who has been granted permanent residence in the Philippines.

A former employee of Scandinavian Airlines System, he is now Manager of M. Y. Travel International Hongkong Ltd., with a monthly salary of P1,200.00. plus allowances. It does not appear that either petitioner has been convicted of a crime involving moral turpitude.

On the other hand, the minor sought to be adopted has been living with them ever since the marriage of petitioners. Petitioner husband has treated the mi -nor as his son, and the latter calls him "Daddy." Although the possibility exists that petitioners may yet have their own children, the adoption at this time, be-fore any such children are begotten, may strengthen, rather than disrupt, future domestic relations.

ISSUE: WON ACQUISITION OF THE NECESSARY TO GRANT ADOPTION. CITIZENSHIP OF THE ADOPTER IS

HELD: NO. The adoption was denied solely because the same would not result in the loss of the minor's Filipino citizenship and the acquisition by him of the citizenship of his adopter.

Unfortunately, the Juvenile and Domestic Relations Court did not expound the reasons for its opinion; but it is clear that, if pursued to its logical conse -quences, the judgment appealed from would operate to impose a further prerequisite on adoptions by aliens beyond those required by law.

As pointed out by the Solicitor General in his brief, the present Civil Code in force (Article 335) only disqualifies from being adopters those aliens that are either(a) non-residents or (b) who are residents but the Republic of the Philippines has broken diplomatic relations with their government. Outside of these two cases, alienage by itself alone does not disqualify a foreigner from adopting a person under our law. Petitioners admittedly do not fall in either class.

Lazatin Vs. Jose Campos (judge)Facts:1.Margarita de Asis died, leaving a holographic will (a will  and testament that has been entirely handwritten and  signed by the testator), providing for a legacy of cash, jewelry, and stocks to respondent Arlene de Leon, a granddaughter; a legacy of support to Rodolfo Gallardo, a son of her late sister; and a legacy of education to Ramon Sta. Clara, son of petitioner Renato Lazatin.

2.During her lifetime, Margarita de Asis kept a safety deposit box at the bank which either she or respondent Nora de Leon (adopted daughter) could open. 5 days after her death, Nora opened the safety deposit box and removed its contents: (a) shares of stock; (b) her adoption papers and those of her sister, respondent Irma L. Veloso; and (c) jewelry belonging to her and to her mother. Nora claims that she opened the safety deposit box in good faith, believing that it was held jointly by her and her deceased mother.

3.Respondents filed a petition to probate (establish the validity) the will. After having learned that the safety box was opened, petitioner's son, Ramon Sta. Clara, filed a motion in the probate court, claiming that the deceased had executed a will subsequent  to that submitted for probate and demanding its pro-duction. He likewise prayed for the opening of the safety deposit box. Respondent Nora L. de Leon admitted that she opened the box but there was no will or any document resembling a will therein.

4.Petitioner Renato to Lazatin alias Renato Sta. Clara filed a motion to intervene in the estate of Margarita de Asis as an adopted child, on the basis of an affidavit executed by Benjamin Lazatin (brother of the deceased Dr. Mariano M. Lazatin), the petitioner was an "illegitimate son" of Dr. Lazatin and was later adopted by him. This affidavit was later modified to state that petitioner was adopted by both Mariano M. Lazatin and his wife Margarita de Asis.

5.During the hearing, Renato presented no decree of  adoption in his favor. Instead, he attempted to prove, over private respondents' objections, that he had recognized the deceased spouses as his parents; he had been supported by them until their death; formerly he was known as "Renato Lazatin" but was compelled to change his surname to "Sta. Clara" when the deceased spouses refused to give consent to his marriage to his present wife; that at first, he and his wife stayed at the residence of Engracio de Asis, father of Margarita, but a few months later, they transferred to the Mercy Hospital at Taft Av-enue, Manila, owned by the deceased spouses, where they continuously resided up to the present. Photographs were also intended to be presented by pe-titioner, e.g., photograph of Irma Veloso where she addressed herself as sister of petitioner; photograph of deceased Margarita de Asis and petitioner when he was a boy; document showing that petitioners real name is "Renato Lazatin."

6.Respondent court first reserved its ruling when petitioner could not present evidence on the issue of his alleged legal adoption, respondent court discon-tinued the hearing and gave the parties time to file memoranda on the question of the admissibility of the evidence sought to be introduced by petitioner.

Issue: Whether or not renato has established his status as an adopted child.

Held: NO.Ratio: Adoption is a juridical act, a proceeding which creates between two persons a relationship similar to that which results from legitimate paternity and filiation. Only an adoption made through the court, or in pursuance with the procedure laid down under Rule 99 of the Rules of Court is valid in this jurisdic-tion. The fact of adoption is never presumed, but must be affirmatively proved by the person claiming its existence. On the contrary, the absence of a record of adoption has been said to evolve a presumption of its non-existence. The absence of proof of such order of adoption by the court, as provided by the statute, cannot be substituted by parol evidence that a child has lived with a person, not his parent, and has been treated as a child to establish such adop-tion. Secondary evidence is nonetheless admissible where the records of adoption proceedings were actually lost or destroyed. But, prior to the introduction of such secondary evidence, the proponent must establish the former existence of the instrument. The correct order of proof is as follows: Existence; exe-cution; loss; contents; although this order may be changed if necessary in the discretion of the court. As earlier pointed out, petitioner failed to establish the former existence of the adoption paper and its subsequent loss or destruction. Secondary proof may only be introduced if it has first beer. established that such adoption paper really existed and was lost.

Republic vs CA and Bobiles GR No. 92326, January 24, 1992

FACTS: Zenaida Corteza Bobiles filed a petition to adopt Jason Condat who had been living with her family since 4 months old. Salvador Condat, father of the child, and the social worker assigned was served with copies of the order finding that the petition was sufficient in form and substance. The copy was also posted on the bulletin board of the court. Nobody appeared to oppose the petition. The judgment declared that surname of the child be changed to “Bobiles”.

ISSUE: WON the petition to adopt Jason should be granted considering only Zenaida filed the petition.

HELD: The petition for adoption was filed when the law applicable was PD 603 (Child and Youth Welfare Code), where such petition may be filed either of the spouses or both of them. After the trial court rendered its favorable decision and while the case was pending on appeal in CA, Family Code took effect where joint adoption of both spouses is mandatory.

Non-joinder is not a ground for the dismissal of an action or a special proceeding. The Family Code will have retrospective application if it will not prejudice or impair vested rights. When Zenaida filed the petition, she was exercising her explicit and unconditional right under said law in force at the time and thus vested and must not be prejudiced. A petition must not be dismissed by reason of failure to comply with law not yet in force and effect at the time. Further -more, the affidavit of consent attached by the husband showed that he actually joined his wife in adopting Jayson. His declarations and subsequent confir -matory testimony in open court was sufficient to make him a co-petitioner. Future of an innocent child must not be compromised by arbitrary insistence of rigid adherence to procedural rules on the form of the pleadings.

Hence, Petition was denied.

Republic vs Claude A. Miller and Jumrus E. Miller G.R. No. 125932. April 21, 1999

Facts: On July 29, 1988, Spouses Miller, both American citizens, filed with the RTC, Angeles City a verified petition to adopt Michael Magno Madayag, a Fi-lipino child, under the provision of the Child and Youth Welfare Code which allows aliens to adopt. The natural parents executed affidavits giving their irre -vocable consent to the adoption and the DSWD recommended approval of the petition on the basis of its evaluation. On May 12, 1989, the trial court rende-red decision granting the petition for adoption.

On August 3, 1998, the Family Code became effective, prohibiting the adoption of a Filipino child by aliens.

The Solicitor General appealed to the granting of the petition for adoption by the RTC.

Issue: Whether or not aliens may be allowed to adopt a Filipino child when the petition for adoption was filed prior to the effectivity of the Family Code prohi -biting the same.

Held: Yes. An alien qualified to adopt under the Child and Youth Welfare Code, which was in force at the time of the filing of the petition, acquired a vested right which could not be affected by the subsequent enactment of a new law disqualifying him.

The enactment of the Family Code, effective August 3, 1988, will not impair the right of respondents who are aliens to adopt a Filipino child because the right has become vested at the time of filing of the petition for adoption and shall be governed by the law then in force. A vested right is one whose exis -

tence, effectivity and extent does not depend upon events foreign to the will of the holder. Vested rights include not only legal or equitable title to the enfor -cement of a demand, but also an exemption from new obligations created after the right has vested.

As long as the petition for adoption was sufficient in form and substance in accordance with the law in governance at the time it was filed, the court acquires jurisdiction and retains it until it fully disposes of the case. To repeat, the jurisdiction of the court is determined by the statute in force at the time of the com -mencement of the action. Such jurisdiction of a court, whether in criminal or civil cases, once it attaches cannot be ousted by a subsequent happenings or events, although of a character which would have prevented jurisdiction from attaching in the first instance.

Therefore, an alien who filed a petition for adoption before the effectivity of the Family code, although denied the right to adopt under Art. 184 of said Code, may continue with his petition under the law prevailing before the Family Code.

Adoption statutes, being humane and salutary, hold the interests and welfare of the child to be of paramount consideration. They are designed to provide homes, parental care and education for unfortunate, needy or orphaned children and give them the protection of society and family in the person of the adopter, as well as childless couples or persons to experience the joy of parenthood and give them legally a child in the person of the adopted for the mani-festation of their natural parent instincts. Every reasonable intendment should be sustained to promote and fulfill these noble and compassionate objec-tives of the law.

Cheesman vs. IAC

(1991)

FACTS: Thomas Cheesman (an American) was married to a Filipina, Criselda. The spouses later separated; but Thomas brought this action to annul the sale of real property made by Criselda in favor of Padilla. He alleged that the sale is void for lack of his consent. The property sold was bought by Criselda using her personal funds, and was registered in her name only.

HELD: The sale was valid. He has no capacity to question the sale of the property by his wife on the theory that in doing so he is merely exercising the pre -rogative of a husband in respect of conjugal property. This would permit indirect controversion of the constitutional prohibition. If the property were to be de-clared conjugal, this would accord to the alien husband an interest and right over the land, which is not granted to him under the Constitution.

G.R. No. 74833 January 21, 1991THOMAS C. CHEESMAN vs. IAC and ESTELITA PADILLA

FACTS: Thomas Cheesman and Criselda P. Cheesman were married on December 4, 1970 but have been separated since February 15, 1981.

On June 4, 1974, a “Deed of Sale and Transfer of Possessory Rights” was executed by Armando Altares conveying a parcel of unregistered land and the

house in favor of “Criselda P. Cheesman, of legal age, Filipino citizen, married to Thomas Cheesman, and residing at Lot No. 1, Blk. 8, Filtration Road, Sta. Rita, Olongapo City.

Thomas Cheesman, although aware of the deed, did not object to the transfer being made only to his wife.

Thereafter, tax declarations for the property purchased were issued in the name only of Criselda Cheesman and Criselda assumed exclusive management and administration of said property, leasing it to tenants. This happened without any protest from Thomas.

Criselda sold the property to Estelita M. Padilla, without the knowledge or consent of Cheesman.

The deed described Criselda as being “of legal age, married to an American citizen...”

Subsequently, Thomas filed a suit in the CFI against Criselda and Estelita Padilla, praying for the annulment of the sale on the ground that the transaction had been executed without his knowledge and consent.

During the Pre-trial, the sale was declared void ab initio and the the delivery of the property to Thomas as administrator of the conjugal partnership property was ordered.

However, the judgment was set aside on a petition for relief filed by the Estrellita, grounded on "fraud, mistake and/or excusable negligence" which had se -riously impaired her right to present her case adequately. Estelita Padilla filed a supplemental pleading as her own answer to the complaint and a motion for summary judgment.

The Trial Court found that — The evidence on record satisfactorily overcame the disputable presumption that all property of the marriage belongs to the

conjugal partnership and that the immovable in question was in truth Criselda’s paraphernal property;

The legal presumption in Article 160 could not apply because the husband-plaintiff is an American citizen and therefore disqualified under the Constitution to acquire and own real properties; and

The exercise by Criselda of exclusive acts of dominion with the knowledge of her husband “had led Estelita to believe that the properties were the exclusive

properties of Criselda and on the faith of such a belief she bought the properties from her and for value ” and therefore, Thomas was estopped to impugn the transfer.

Thomas appealed the judgment, as well as the act of the Trial Court of granting Estelita ’s petition for relief and its resolution of matters not subject of said petition.

IAC affirmed the Summary Judgment and found no reversible error. Thomas Cheesman appealed to the Supreme Court.

ISSUE: Whether or not Thomas correctly availed of the remedy of appeal to SC? –

RULING: An order of the CFI granting a petition for relief under Rule 38 is interlocutory and is not appealable.

QUESTION OF FACT vs. QUESTION OF LAW:

The conclusions made by the trial court were derived from evidence adduced by the parties, the facts set out in the pleadings or otherwise appearing on re-cord—are conclusions or findings of fact

As distinguished from a QUESTION OF LAW —which exists "when the doubt or difference arises as to what the law is on a certain state of facts" — "there is a

QUESTION OFFACT when the doubt or difference arises as to the truth or the falsehood of alleged facts;" or when the "query necessarily invites calibration of the whole evidence considering mainly the credibility of witnesses, existence and relevancy of specific surrounding circumstances, their relation; to each other and to the whole and the probabilities of the situation."

The RULE is that only questions of law, distinctly set forth, may be raised in a petition for the review on certiorari of a decision of the Court of Appeals pre-sented to the Supreme Court.

The appellate jurisdiction of the SC is limited to reviewing errors of law, accepting as conclusive the factual findings of the lower court upon its own assess -ment of the evidence.

CA was created precisely to take away from the SC the work of examining the evidence, and confine its task to the determination of questions which do not call for the reading and study of transcripts containing the testimony of witnesses. The rule of conclusiveness of the factual findings or conclusions of the CA is subject to certain exceptions. However, none of which is present in the case at bar.

HELD: Section 14, Art. XIV of 1973 Constitution provides

that: ―save in cases of hereditary succession, no private land shall be transferred or conveyed except to individuals, corporations, or associations qualified

to acquire or hold lands of the public domain.‖

Thus, assuming that it was his intention that the lot in question be purchased by him and his wife, he acquired no right whatsoever over the property by vir-tue of that purchase; and in attempting to acquire a right or interest in land, he was knowingly violating the Constitution. As such, the sale to him was null and void. At any rate, Cheesman had and has NO CAPACITY TOQUESTION THE SUBSEQUENTSALE OF THE SAME PROPERTY BY HIS WIFE ON THETHEORY THAT IN SO DOING HEIS MERELY EXERCISING THE PREROGATIVE OF AHUSBAND IN RESPECT OFCONJUGAL PROPERTY. To sustain such a theory would permit in direct controversion of the Constitutional prohibition.

If the property were to be declared conjugal, this would accord to the alien husband a not in substantial interest and right over land, as he would then have a decisive vote as to its transfer or disposition. This is a right that the Constitution does not permit him to have. Even if the wife did use conjugal funds to make the acquisition, his recovering and holding the property cannot be warranted as it is against the constitution. Consequently, Estelita is a purchaser in good faith since she knew that Thomas cannot intervene in the sale or disposition of the said property. DECISION: The Court AFFIRMED the appealed de -cision.

Mathews vs Taylor 590 SCRA 394

Facts: On June 30, 1988, respondent Benjamin A. Taylor (Benjamin), a British subject, married Joselyn C. Taylor (Joselyn), a 17-year old Filipina. On June 9, 1989, while their marriage was subsisting, Joselyn bought from Diosa M. Martin a 1,294 square-meter lot (Boracay property) situated at Manoc-Manoc, Boracay Island, Malay, Aklan, for and in consideration of P129,000.00.

The sale was allegedly financed by Benjamin. Joselyn and Benjamin, also using the latter ’s funds, constructed improvements thereon and eventually converted the property to a vacation and tourist resort known as the Admiral Ben Bow Inn. All required permits and licenses for the operation of the resort were obtained in the name of Ginna Celestino, Joselyn’s sister.

However, Benjamin and Joselyn had a falling out, and Joselyn ran away with Kim Philippsen. On June 8, 1992, Joselyn executed a Special Power of Attor -ney (SPA) in favor of Benjamin, authorizing the latter to maintain, sell, lease, and sub-lease and otherwise enter into contract with third parties with respect to their Boracay property. On July 20, 1992, Joselyn as lessor and petitioner Philip Matthews as lessee, entered into an Agreement of Lease (Agreement) involving the Boracay property for a period of 25 years, with an annual rental of P12,000.00.

The agreement was signed by the parties and executed before a Notary Public. Petitioner thereafter took possession of the property and renamed the re -sort as Music Garden Resort. Claiming that the Agreement was null and void since it was entered into by Joselyn without his (Benjamin’s) consent, Benja-min instituted an action for Declaration of Nullity of Agreement of Lease with Damages against Joselyn and the petitioner.

Benjamin claimed that his funds were used in the acquisition and improvement of the Boracay property, and coupled with the fact that he was Joselyn ’s husband, any transaction involving said property required his consent.

Issue: Can an alien husband nullify a lease contract entered into by his Filipina wifebought during their marriage?

Held: The rule is clear and inflexible: aliens are absolutely not allowed to acquire public or private lands in the Philippines, save only in constitutionally reco -gnized exceptions.

There is no rule more settled than this constitutional prohibition, as more and more aliens attempt to circumvent the provision by trying to own lands through another. In a long line of cases, we have settled issues that directly or indirectly involve the above constitutional provision.

We had cases where aliens wanted that a particular property be declared as part of their father ’s estate; that they be reimbursed the funds used in purcha-

sing a property titled in the name of another; that an implied trust be declared in their (aliens ’) favor; and that a contract of sale be nullified for their lack of consent. Benjamin has no right to nullify the Agreement of Lease between Joselyn and petitioner. Benjamin, being an alien, is absolutely prohibited from acquiring private and public lands in the Philippines.

Considering that Joselyn appeared to be the designated “vendee” in the Deed of Sale of said property, she acquired sole ownership thereto. This is true

even if we sustain Benjamin’s claim that he provided the funds for such acquisition. By entering into such contract knowing that it was illegal, no implied trust was created in his favor; no reimbursement for his expenses can be allowed; and no declaration can be made that the subject property was part of the conjugal/community property of the spouses.

In any event, he had and has no capacity or personality to question the subsequent lease of the Boracay property by his wife on the theory that in so doing, he was merely exercising the prerogative of a husband in respect of conjugal property. To sustain such a theory would countenance indirect controversion of the constitutional prohibition. If the property were to be declared conjugal, this would accord the alien husband a substantial interest and right over the land, as he would then have a decisive vote as to its transfer or disposition. This is a right that the Constitution does not permit him to have.